Problemas de algebra, geometria y trignonometria 9789703604685, 9789703604692

PRESENTACION El presente libro contiene 658 problemas, minuciosamente resueltos, en el dominio de las matematicas elemen

119 8 48MB

Spanish Pages 401 Year 2010

Report DMCA / Copyright

DOWNLOAD PDF FILE

Table of contents :
PROBLEMAS DEA.LGEBRA, GEOMETRlA YTRIGONOMETRlA
PÁGINA LEGAL
INDICE
PRESENTACION
ALGEBRA
1. Progresiones aritmetica y geometrica
Observaciones preliminares
2. Ecuaciones algebraicas y sistemas de ecuaciones
3. Desigualdades algebraicas
4. Ecuaciones logaritmicas y exponenciales, identidades y desigualdades
5. Combinatoria y binomio de Newton
6. Planteamiento de ecuaciones
7. Problemas diferentes
GEOMETRIA
A. PLANIMETRiA
1. Problemas de calculo
2. Problemas de construccion
3. Problemas de demostracion
4. Lugar geometrico de los puntos
5. Determinacion de los valores maxim os y minimos
B. ESTEREOMETRIA
1. Problemas de calculo
2. Problemas de construccion
3. Problemas de demostracion
4. Lugar geometrico de los puntos
5. Determinacion de los valores maxim os y minimos
TRIGONOMETRIA
1. Transformaci6n de las expresiones que contienen funciones trigonometricas 533.
2. Ecuaciones trigonometricas y sistemas
3. Funciones trigonornetricas inversas
4. DesiguaJdades trigonornetricas
5. Problemas diferentes
ALGEBRA
RESOLUCIONES Y SOLUCIONES
1. Progresiones aritmetica y geometrica
Recommend Papers

Problemas de algebra, geometria y trignonometria
 9789703604685, 9789703604692

  • 0 0 0
  • Like this paper and download? You can publish your own PDF file online for free in a few minutes! Sign Up
File loading please wait...
Citation preview

COLECCIÓN CIENCIAS BÁSICAS

PROBLEMAS D E ÁLGEBRA, GEOMETRÍA Y TRIGONOMETRÍA A. NlETUSHIL L

TALA^ JCCION: GUILLERMO GARCÍA TA LAVE fERA I :

ti

STITUTO POLITÉCNICO NACIONAL

COLECCION CIENCIAS BAsICAS PROBLEMAS DEA.LGEBRA, GEOMETRlA YTRIGONOMETRlA A. Nietushil

DIRECTORIO IPN

DIRECTORIO UTP

DR. JOSE ENRIQUE VILLA RIVERA Director General

M. EN C. GERARDO MARCELINO LARA OROZCO Rector

DR. EFREN PARADA ARIAS Secreta rio General

M. EN C. LUIS RICARDO MORALES JUAREZ Secrctario Academico

ORA. YOLOXOCHITL BUSTAMANTE DIEZ Secretaria Academica

QFB. MARIA ONEIDA ROSADO GARCIA Secretaria de Vinculacion

DR. JOSE MADRID FLORES Secretario de Extension e Integraci6n Social DR. LUIS HUMBERTO FABILA CASTILLO Secretario de Investigacion y Posgrado DR. HECTOR MARTINEZ CASTUERA Secretario de Servicios Educativos

L1C. I'CO. JAVIER TREVINO MUSALEM Abogado General CPo MONICA CANDIA DE LA ROSA Contralora Interna CPo TOMAs E. SILVA LIMON Director de Administraci6n y Finanzas

DR. MARIO ALBERTO RODRiGUEZ CASAS Secretario de Administracion

DIRECTORES DE CARRERA:

Lie. LUIS ANTONIO RIOS CARDENAS

CPo JOSE DONATO RODRiGUEZ ROMERO Administracion

Secretario Tecnico ING. LUIS EDUARDO ZEDILLO PONCE DE LEON Secretario Ejecutivo de la Comision de Operacion y Fomento de Actividades Acadell1icas ING. JESUS ORTIZ GUTIERREZ Secretario Ejecutivo del Patronato de Obras e Instalaciones MTRO. FERNANDO SARINANA MARQUEZ Director de XE-IPN TV Canal I I

ING. MARCOS ESPINOSA MARTiNEZ Electricidad y Electronica Industrial ING. EDGAR EDUARDO PASTOR PEREZ Mantcnimiento Industrial M. EN C. RAMON BELTRAN MARTINEZ Mecatronica ING. JOSE BELARMINO BUENO MONEDA Procesos de Produccion

Lie. LUIS ALBERTO CORTES ORTIZ Abogado General

Lie. ARTURO SALCIDO BELTRAN Director de Publicaciones

LlC. LEONEL TORRES LEZAMA Tecnologias de la Informacion y Comunicaci6n ING. POMPILIO MARTiNEZ VALENCIA Encargado de Tecnologia Ambiental

COLECCION CIENCIAS BAsICAS PROBLEMAS DE ALGEBRA, GEO~TRiA YTRIGONO~TRiA

A. Nietushil

Compilaci6n y Traducci6n. Guillermo Garcia Talavera

Instituto Politecnico Nacional -Mexico--

Coleccion Ciencias Basicas Problemas de Algebra, Geometria y Trigonometria A. Nietushil Traductor: Guillermo Garcia Talavera Primera Edicion: 2008 D.R. © 2008 Instituto Politecnico Nacional Luis Enrique Erro sin Unidad Profesional "Adolfo Lopez Mateos" Col. Zacatenco, 07738, Mexico D.E Direccion de Publicaciones Tresguerras 27, Col. Centro Historico 06040, Mexico D.E http:/www.publicaciones.ipn.mx ISBN Coleccion: 978-970-36-0468-5 ISBN: 978-970-36-0469-2 FIPN: 2007-505 Impreso en Mexico/Printed ill Mexico

PRESENTACION

El presente libro contiene 658 problemas, minuciosamente resueltos, en el dominio de las matematicas elementales: Algebra, Geometria (plana y del espacio) y Trigonometria. Constituye, por tanto, un recurso valioso para la preparaci6n de aspirantes a ingresar a escuelas de educaci6n media superior y superior. Se trata de la traducci6n de un texto originalmente escrito en lengua rosa, destinado a reforzar el instrumental materruitico basico de los alurnnos de instituciones denominadas Tejnikum, cuyos curricula corresponden, en sus etapas iniciales, a los de los CECyT del IPN y, en las finales, a los de las UT mexicanas. Esta coedici6n IPN-UTP, hani posible poner al alcance econ6mico de los alurnnos de estas instituciones este valioso aporte bibliognifico, que los ayudani a la comprensi6n de los materiales matematicos que se exponen en cursos mas avanzados: GeometriaAnalitica y Calculo Diferencial e Integral. Esta es una de las mas valiosas traducciones, entre mas de cien que ha publicado el ilustre maestro Guillermo Garcia Talavera.

ALGEBRA

,

9

Tambien forman una progresion aritmetica.

ALGEBRA

2. Los numeros positivos aI' a 2, aritmetica. Demostrar que

1. Progresiones aritmetica y geometrica

_

Observaciones preliminares Si an es el tennino enesimo, d-Ia diferencia y Sn-la suma de los n primeros terminos de una progresion aritmetka, entonces

~l~ __

JQ1 +,Ja2

+

1

,Ja2 + JQ1

••• ,

+ + .. .

an

forman una progresion

1

~ an-I +.ra;;

=

n-l

JQ1 +.ra;; .

3. Demostrar que si los numeros a" a 2, ••• , an no son iguales a cero y forman una progresion aritmetica, entonces

(1)

= (a\+a n ) = 12a\+d(n - l) ln

S

2

n

2

(2)

Si u, es el termino enesimo, q ---el denominador y Sn-Ia suma de los n primeros terminos de una progresion geometrica, entonces n-l

= u1q S = unq-ul = uI(qn_l) un

n

q-l

q-l

4. Demostrar que toda sucesion de numeros a" a 2, para cualquier n ~ 3 satisfacen la condicion

(4)

5. Mostrar que para toda progresion aritmetica aI' a 2, a 3 • • • , an'

(5)

1. Demostrar que si los numeros positivos a, b, c forman una progresion aritmetica, los nlimeros 1

1

1

.Jb+JC' JC+.Ja' .Ja+.Jb

que

forma una progresion aritmetica.

tienen lugar las igualdades a 1 - 2a 2 +a 3=0,

+ 3a 3-a 4 = 0, 4a 2 + 6a 3 -4a 4 + a 6 = 0,

a 1 - 3a 2 l-q

an'

(3)

Si, por fin, S es la suma de una progresion geometrica decreciente infmita (iqi< I), entonces S=~

••• ,

a1 -

y que, en general, para cualquier n>2 tenemos

10

PROBLEMAS DE MATEMATICAS ELEMENTALES

Indicacion. En este y en el siguiente problema es conveniente hacer uso de la identidad facil de comprobar

6. Demostrar que para cualquier progresion aritmetica a" ... , an' an+I , ••• siendo n ~ 3 tiene lugar la igualdad

a

12 -

(~ ) a; + ... + (- 1Y (~ ~; + 1 = O.

7. Demostrar que si los numeros klog x, mlog x, nlog x (x forman una progresion aritmetica, entonces n2

"* 1)

= (1m )kIOgm.

8. Hallar una progresion aritmetica en la que la relacion entre la suma de los n primeros terminos y la suma de los 1m siguientes no depende de n. 9. Los numerosxI,x2, •• • ,xn forman una progresion aritmetica. Hallar esta progresion, si XI

+ X 2 + ... + Xn

=

a,xI2 + X 22 + ... + Xn3 = b3 .

Indicacion. En este y en el siguiente problema es conveniente

hacer uso de la igualdad 12 + 22 + 32 + ... + n 2

= n (n+I)(2n+l) 6.

10. La sucesion de nllineros 1,4, 10, 19, ... posce ~ propiedad de que la diferencia de dos numeros vecinos forman una progresion aritmetica. Hallar el termino enesimo y la suma de los n primeros terminos de esta sucesion de nllineros. 11. Hagamos una tabla

1, 2,3,4 3, 4, 5, 6, 7

4,5,6, 7, 8, 9, 10 Demostrar que la suma de los terminos de cada fila es igual al cuadrado de un numero impar. 12. En la progresion geometrica aI' a 2, a3 , ••• se conocen los terminos a m+n = A, a m-n = B. Hallar a m y an (A*- 0). 13. Supongamos que Sn es la suma de los n primeros terminos de una progresion geometrica, Sn "* 0, q"* 0. Demostrar que

14. Conociendo la suma Sn de los n primeros terminos de una progresion geometrica y la suma de las reciprocas de estos terminos, hallar el producto Pn de los n primeros terminos de la progresion. 15. Hallar la suma 1+2x+3x2 + 4x 3 + ... +(n+lh n .

ALGEBRA

Demostrar que xI' x 2 ' ••• , x n' ••• son terminos sucesivos de una progresion geometrica. lndicaciones. Se puede aplicar el metodo de induccion completa.

16. Hallar la suma 1+11+111+ ... +111 ... 1, si e1 ultimo sumando es un nillnero de n cifras.

22. Se conocen una progresi6n aritmetica con el termino comtIn an y una progresion geometrica con el termino comtIn bn, con la particularidad de que para todos los numeros naturales de n, . al = bl , a 2 = b2 , a l "* a 2 Y an > O. Demostrar que para n>2, a nO.

2,}

u2 + v 2 + W = v2 + w 2 +u = 2,

61. Hallar las soluciones reales del sistema de ecuaciones

w 2 + u 2 +v = 2.

6,1 I xz + yz = (xy + 1tJ +y +z = X2 + y2 + Z2 = 14, X

58. Resolver el sistema de ecuaciones

2}

X- +xy + y = 1, ?

?

2

?

2

X- +XZ+z = 4,

62. Resolver el sistema de ecuaciones

Y + yz+z = 7.

a'l}

x 2 +xy+xz + yz = y2 + xy + xz + yz = b,

I

59. Resolver el sistema de ecuaciones

~=a Xl

Z2

XIX:~.. XIJ = Q2'

suponiendo que abc:f::.

. ... ~ ......... .

x,x, . X,,_, Xl

_ -

+ xy + xz + yz = c,

I'

an'

j

suponiendo que los numeros aI' ... , anY Xl'

o.

63. Resolver el sistema de ecuaciones

... ,

60. Resolver el sistema de ecuaciones

(X+ y+z)(ax+ y+z)=e,l

(x + y + z) (x + ay + z) = [2, ~ (x + y + z) (x + y + az) = m2 , J

xn son positivos.

suponiendo que abc

= O.

64. Hallar las soluciones reales del sistema de ecuaciones

i Z3

+Z3 =2a(yz+zx+xy ),1 + x 3 = 2b (yz + zx + xy),

I

x 3 + i = 2c(yz +zx +xy).j

ALGEBRA

65. Resolver el sistema de ecuaciones

y + 2x + z

69. Resolver el sistema de ecuaciones

= a{x + y)(z + x),l

z+2y+x=b(y+z)(x+ y),(.

x- + y

x+2z+ y=c(z+x)(y+z)J

v

=9,}

xy + xz + yz

z

?

I

70. Sean (x,y, z) las soluciones del sistema de ecuaciones:

~+~+~=1 x

2

3 x 3 +y 3 +z 3 =a. , -'

66. Resolver el sistema de ecuaciones

x+ y+z

= a,1 + z- = a 2 ,).I

y+ y + z ?

x+ y+z =a, x 2 + y2

'

= 27.

=b 2 ,

+Z2

1+1+1=1. x

67. Resolver el sistema de ecuaciones

y

:

c

Hallar la suma X

x+ y+z=a,1

xy+ yz +xZ

~ a:.l

xyz

= a .J

3

3

+ Y +z

3

71. Resolver el sistema de ecuaciones

68. Demostrar que la soluci6n (mica de sistema de ecuaciones

0'1

2x+ Y + z = yz+zx+xy=0,

i

xy+ es la soluci6n x = y = z = 0.

Z2

I

= o.J

72. Resolver el sistema de ecuaciones

17

18

PROBLEMAS DE MATEMATICAS ELEMENTALES

,

73. Resolver el sistema de ecuaciones

I + x + y + z == 0, I a + bx + cy + dz ==

xy + yz + zx == 47,1 x 2 + y2 ==

Z2,

~

(z - x) (z - y) = 2.j

2

')

')

2

OJ

I

a +b-x+c-y+d z==O.)

entonces la multiplicaci6n xyz es positiva.

74. Hallar todas las soluciones reales del sistema de ecuaciones

En las ecuaciones propuestas a continuaci6n, en el caso de raices cuyas potencias sean pares, se examinan s6lo aquellos valores de las inc6gnitas para los cuales la expresi6n contenida bajo la raiz no es negativa; tomando, al mismo tiempo, solamente los valores no negativos de la raiz. En el caso de rakes impares, la expresi6n contenida bajo la raiz puede ser cualquier numero real (en este caso el signo de la rafz coincide con el signo de la expresi6n subradical).

75. Hallartodas las soluciones reales del sistema de ecuaciones

77. Resolver la ecuaci6n

78. Resolver la ecuaci6n m!(

Ijl-x

)2

m l-( -)2 m j 2 - 'l/ l-x ="I-x.

79. Resolver la ecuaci6n

76. Demostrar que si a, b, c y d son por pares numeros reales desiguales y x,yy z son las soluciones del sistema de ecuaciones

80. Resolver la ecuaci6n

r -;:.

fx---;~Jx - -J~ - -l~ == l, 2 V x+ ,/ x

ALGEBRA

19

86. Resolver el sistema de ecuaciones

81. Resolver la ecuaci6n ~ ~ 7 IX~X +'\/x+7 = - - - .

~ ~x+1

~x+l

82. Hallar todas las rakes reales de la ecuaci6n 87. Resolver el sistema de ecuaciones

/Y+I +2\j~ =3 1 y+1 '~

'\ x-y

83. Resolver la ecuaci6n

x+xy+y=7.1

~x -4a+ 16

)

2a + 4 --E.

= 2~x -

88. Hallartodas las soluciones reales del sistema de ecuaciones LPara cwiles valores reales de a tendra soluci6n la ecuaci6n? 84. resolver el sistema de ecuaciones

)t

x+ y - -,,/I'~ . x- y -

I~}

x- y ,

xy = IS.

-v'I 1-16y 2 - '.i'"1-16x~~ =2 ( x+ y, X

2

~

1

+ y~ + 4xy = s.J

85. Resolver el sistema de ecuaciones

312)}

x - y ="27('/'2 ~, X Y - 'V xy ,

if;-VY=3.

89. Resolver el sistema de ecuaciones

Y+

2~ x~12 y + I = x 2 + 17 f 3

12

'

x -2 = /IX -+ --'- I- - -Y 8y 3 'v3y'4 2x·J

20

PROBLEMAS DE MATEMATICAS ELEMENTALES

95. Resolver el sistema de ecuaciones

90. Resolver el sistema de ecuaciones

X+~X2 _y2

-~'-;== 2 ~2 x- x -y

17}

x_~x2 _y2 + - 1- 2 2 = -4 ' X+ , x -y

x-Jx - y~y = a(-Jx -~),} (a>O,b > O). ??

1

x- + xy+ y = b-

x(x+ y)+ -Jx 2 +xy+4 =52. 91. Resolver el sistema de eeuaeiones

l

+ ~3l-2x+3 = tx+ 5,} 3x-2y=5.

3. Desigualdades algebraicas Observaciones preliminares Expongamos algunas desigualdades utilizadas en la resoluei6n de los problemas que se proponen mas abajo. Para cualesquiera a y b reales

92. Hallar las soluciones reales del sistema de eeuaeiones

(I) La desigualdad (1) es una deduce ion de la evidente desigualdad (a ± b)2 ~ EI signa de igualdad en (1) tiene lugar solo en el easo en que lal=lbl. Si ab > 0, dividiendo las dos partes de la desigualdad (1) entre ab se tendra:

°.

93. Resolver eI sistema de ecuaeiones

(x-YNY=~,l (x+ y}f; =3-jyJ 94. Resolver el sistema de eeuaeiones

x

~ +y - ~

x- a,t

y = ,- (a > O). /22 ' 222 \Ix +y +ljx -y =a J

a

b a

-+ - ~2.

b

(2)

Si u~O, v~O, entonees, haciendo en (1) u = a2, v = b2 , obtendremos

u+v

,

-- ~~uv.

2

(3)

En las desigualdades (2) y (3) el signa de igualdad tiene lugar solamente cuando a = b y u = V, eorrespondientemente.

ALGEBRA

Sefialemos, ademas, ciertas propiedades del trinomio cuadrado 7 (4) Y = ax- + bx + c, que en adelante se emplean en toda una serie de problemas. De la representaci6n del trinomio (4) par la f6rmula b)2

y=a ( x+ -

20

-

2i

Para todos los demas valores de x diferentes de XI y x 2' el trinomio po see el mismo signa que a. Asi pues, el trinomio siempre conserva el signo del coejicienle del termino de mayor polencia, excepto en el caso en que sus raices son reales y Xl ~ X ~

x2 •

96. Hallar todos los valores reales de r para los cuales el polinomio

b 2 - 4ac 4a

(5)

se deduce que en el caso cuando el discriminante del trinomio es positivo para todos los valores reales dex. (en este caso las rakes del trinomio no son reales), el trinomio adquiere para todos los valores dex, val ores de un mismo signo, que coincide con el signa del coeficiente a del termino de mayor potencia. En el caso en que D = 0 el trinomio conserva tambien el signo constante, haciendose igual a cero para el valor unico x

=-

;0 .

Por fin, cuando D > 0 (en este caso, las rakes x I y x 2 del trinomio son reales y diferentes), de la descomposici6n

97. Demostrar que la expresi6n (? v~ '\ ( y) 3l2-.+"-- 1-8 -"- + - + 10 )1

2

X

2)

).

X

no es negativa para cualesquiera X e y reales y no iguales a cero.

98. (,Para cuales valores de a se satisface el sistema de desigualdades

cualesquiera que sean los valores de x? se deduce que solamente con la condici6n de que

99. Demostrar que para cualesquiera valores reales de los numeros a, b, c y des valida la desigualdad el trinomio adquiere valores de signo contrario al de a.

22

PROBLEMAS DE MATEMATICAS ELEMENTALES

100. Hallar todos los valores de a para los cuales el sistema rrusmo x2 +

+ 2x ~

l

105. Demostrar que siendo n> 2

I,}

x-y+a=O tiene soluci6n (mica. Hallar las soluciones correspondientes.

106. Demostrar que con tres segmentos de longitud a> 0, b > 0 y c > 0 se puede construir un triangulo solamente cuando 2

?

?

pa- + qb > pqc101. Hallar los pares de los numeros enterosxe y que satisfacen al sistema de desigualdades y-i x 2

-

para cualesquiera numeros p y q enlazados en la relaci6n

a,}

1

2x I + >

p+q=l.

y+ i x-ll< 2.

107. Demostrar que para cualesquiera valores reales de x, y y z es valida la desigualdad

102. Demostrar que para cualquier valor entero de n > 1 es valida la desigualdad

1

1

I

n+ 1

n+ 2

2n

1 2

- - + -- + ... +-- >-. 103. Demostrar que para cualquier valor entero y positivo de m es valida la desigualdad 1

1

1

»l.

- + - - + ... + ( m +1 m +2 m + 2m + 1

104. Demostrar que para cualquier valor entero positivo de n I

lin-I

'2" + -3 2 +"'+ -n 2 0 para que siendo R 2 r > 0 sea valida la desigualdad 2

2

?

0< ,-1 +R_~~ < I? 2dR _.

-----~---

-

- --

ALGEBRA --

23

117. Demostrar que

111. Demostrar la desigualdad

Ixlal +xZa2 + ... +xnan !: 0, a -::j. 1. A continuacion, al resolver toda una serie de problemas, para pasar de los logaritmos con base a a los logaritmos con base b y viceversa, se usa la formula siguiente

... , anY bl' b2 ,

a~ + a~

+ ... +

a~ =

(II

I,}

og

N

b

logN

(2)

h loga

(esta formula se demuestra por logaritrnacion de la identidad (1) tomando como base b). De la formula (1), siendo N=b, en particular, se deduce:

2 2 b22+···+ bn=l. b1+

es valida la desigualdad

I. Hallar estaraiz.

129. Resolver la ecuaci6n ,,'.[; log a

,

d\oga

1+

a

log(p - x) = 2j

a log(x

u,

1-

+ "loga· "log2x=O.

130.;.,A cuales condiciones deberan satisfacer los numeros a y b para que la ecuaci6n

xy 10gb = ~ logx

tenga por 10 menos una soluci6n? Hallartodas las soluciones de esta ecuaci6n.

131. Resolver laecuaci6n*l

FIog~+xlog~

-

-q

log 4 p-q log(x + q)

q)

(p>q>O).

134. Resolver la ecuaci6n

-i5 log

1+hlog(210ga -

XI)

x

log 5-J 5 + -i5 log 515

= -16.

135. Resolver laecuaci6n

136. Resolver la ecuaci6n

GCuantas ralces tiene esta ecuaci6n para un valor determinado de n?

137. Resolver la ecuaci6n I·

+

25

,j

(j

I

10g~~+xlogV¥ = a.

log 2·sen 2 x loga + 1 = O.

138. Resolver el sistema de ecuaciones

132. Resolver la ecuaci6n

log(~ + 1) =3 logVx-40

senx

.

*) Aquf y a cotinuaci6n las rafces se comprenden en el sentido mencionado en la pagina 24.

26

PROBLEMAS DE MATEMATICAS ELEMENTALES

139. Resolver el sistema de ecuaciones XO

= yb'1

Cloa.,!.= .Clogx to Y C logy

(a '" b, ab '"

O.)J

140. resolver e\ sistema de ecuaciones

= 7,1 v 12 X' = 5 .j

61ogx+38logy

141. Resolver e\ sistema de ecuaciones

yxylogx= X2, 5 } 410g y-Y log (y - 3x) = 1. 142. Resolver el sistema de ecuaciones 1

aXb Y =ab,1

2° logx=b logy· ""1ogb.J

144. Resolver el sistema deecuaciones

1210gx(_I_+210gy1=2\OgX,l ~ Xlog2

2\ogx . 3\og (x + y) =3 3 \ogx.J 145. Resolverel sistema de ecuaciones

x2logy~ log 2 ~ yjY ~- xlog 2),} yO

2y2

xy = 81.

log2-.J21 ogx = 1.

146. Resolver el sistema de ecuaciones 2

4

~

4

logx+ logy+ logz = 2, I

2'J

31ogy+9Iogz+91ogx = 4Iogz+16\ogx+16 1ogy = 2. 147. Resolverel sistema de ecuaciones

, log ( y - x ) + ~~ log y1 = -2, ~I

05

~

143. Resolver el sistema de ecuaciones

3( iOgX-~IOgy) = 10,}

J

~

x- + y148. Resolver la ecuaci6n

= 25.)I

ALGEBRA

27

153. Resolverel sistema de ecuaciones

149. Resolver el sistema de ecuaciones

suponiendo que x > 0, y > 0 y pq > O.

suponiendo que x > 0 y y > o.

154. Resolver el sistema de ecuaciones 150. Resolver el sistema de ecuaciones a 2x

2b,}

+ a 2y == a X + Y == c

(a>

0). suponiendo que x > 0, y > 0, p > 0, q> O.

GA cmiles condiciones debenin satisfacer bye para que el sistema tenga soluci6n?

155. Demostrar que

151. Hallar las soluciones del sistema de ecuaciones si a2 + b2 == c2 y a> 0, b> 0, c> O.

156. Simplificar la expresi6n

(hloga-alogb) + (b±IOga_a' 10gb) + ... suponiendo que x > 0, y > 0 y n > 0.

... + (

b-j;;

loga-

02'1

]2

10gb .

152. Resolver el sistema de ecuaciones

(3x + yy-y== 9,} x y

,---

- ~324 ==18x 2 +12xy+2/.

157. Simplificar la expresi6n a '0',:,':'" ,admitiendo que todos los logaritmos han sido tornados con la misma base b.

28

PROBLEMAS DE MATEMAnCAS ELEMENTALES

158. Se conoce: alogb = A, qlogb = By un numero entero n > O. Calcular el Clog b, donde c es igual al producto de n terrninos de una progresi6n geometrica con eJ primer terrnino a y el denominador q. 159. Demostrar que si para cierto valor positivo de N t= 1, para los tres numeros positivos a, bye se cum pie la relaci6n UlogN

GlogN_blogN

ClogN

blogN-ClogN'

163. Resolver la desigualdad

164. Resolver la desigualdad

°logx+Olog(x + 1) I).

165. Resolver la desigualdad 3 log (x 2

entonces, b es el valor medio proporcional entre aye y la relaci6n se cumple para cualesquiera val ores positivos de N t= 1 .

-5x+6) 0, q> O.

238. Las rakes XI y x 2 de la ecuaci6n Xl - 3ax + a 2 = son tales que xf + xi = 1,75. Determinar a.

243. Demostrar que si ambas rakes de la ecuaci6n

239. Se tiene la ecuaci6n X2 + px + q = O. Componer una ecuaci6n cuadnitica, cuyas rakes sean

son positivas, entonces las rakes de la ecuaci6n

X2

+ px+ q = 0

qi + (p - 2rq) y + I - pr

=

°

senin tambien positivas para todos los valores de r ;::: O. Aclarar si es justa esta afirmaci6n siendo r < O.

ALGEBRA

244. Hallar todos los valores reales de p para los cuales las rakes de fa ecuaci6n (p - 3 )X2 - 2 px + 6 p = 0 son reales y positivas.

245. Para cualquier valor positivo de Ie todas las rakes de la ecuaci6n

ax2 + bx + c + Ie = 0 son reales y positivas. Demostrar que en este caso a = 0 (se supone que los coeficientes a, by c son reales).

246. Demostrar que ambas ralces de la ecuaci6n Xl

+X + I=0

39

248. Demostrar que para cualesquiera valores reales de a,p y q, las ralces de la ecuaci6n 1 - -+ - - =-0 x- p x-q ason reales. 249. Demostrar que la ecuaci6n cuadnitica

a 2 x 2 + (b 2 + a 2 - c 2 )x + b2 = 0 no puede tener ralces reales si a + b > c y la-hi < c.

250. Se conoce que Xl' x 2 y X 3 son las ralces de la ecuaci6n

x3 -

2X2

+ X + 1 = O.

Componer una nueva ecuaci6n, cuyas rakes sean los numeros

satisfacen a la ecuaci6n

251. Se conoce que XI' x 2 YX3 son las ralces de la ecuaci6n donde m, n y p son numeros enteros cuafesquiera.

-l-

3

-X

2

-1=0

Componer una nueva ecuaci6n, cuyas rakes sean los numeros

247. EI sistema de ecuaciones

a(x2

X

i)+ X + Y - Ie = 0,1 X-Y+Ie=oJ

tiene soluciones reales para cualquier valor de Ie. Demostrar que a = 0

252. Expresar ef termino independiente c de la ecuaci6n cubica

40

PROBLEMAS DE MATEMATICAS ELEMENTALES

por medio de los coeficientes a y b, conociendo que las rakes de la ecuaci6n forman una progresi6n aritmetica.

256. Todas las rakes del polinomio p(x)=x 3 + px+q

253. Supongamos que todas las rakes de cierta ecuaci6n x 3 + px 2 + qx + r = 0 sean positivas. ~A cwil condici6n suplementaria debenin satisfacer sus coeficientes p, q y r para que de los segmentos, cuyas longitudes son iguales aestas rakes, se pueda construir un triangulo?

con coeficientes rea!es y q ::/; 0 son reales. Demostrar que P < o. 257. Demostrar que la ecuaci6n

donde ay b son reales y b > 0, tiene solamente una raizpositiva. 258. Hallar todos los val ores reales de a y b para los cuales las ecuaClones

lndicacion. Estudiar la expresi6n

,

2

XO

254. Las ecuaciones X

3

+ ax + 18 = 0,

x 3 + bx + 12 =

+ Plx+ql =0,

x 3 + P2 X + q2

=0

°

tienen dos rakes comunes y determinar estas rakes. 259. Demostrar que

(PI ::/; P2' ql ::/; q2) tienen una raiz comun. Hallar esta raiz y las demas rakes de ambas ecuaciones.

",120+14"' 2 + 1/ 20-14\1'2 =4.

255. Hallar todos los val ores de 'A para los cuales las ecuaciones

260. Supongamos que a, bye sean por pares numeros no iguales entre sl. Demostrar que la expresi6n

( ) =0 "'--'< 3 -x 2 -x-/dl

31

'Ax2-x-('A+l)=0 tienen una raiz com un y hallar esta raiz.

no es igual a cero.

r-

3/

r;:;

ALG EBRA

261. Descomponer en factores la expresi6n ( x+y+z ) 3 - x3 - y3- z 3 ;

262. Demostrar que si tres numeros reales a, bye esta enlazados por la relaci6n 1 1 1

-+ - + - = - -- abc

a+b+c

entonces, obligatoriamente dos cualesqu iera de estos numeros son iguales en valor absoluto y tienen signos contrarios. 263. Determinar para cuales valores complejos de p y q el binomio X4 - 1 se divide entre el trinomio cuadrado x 2 + +px +q.

264. l,Para cuales valores de a y n el polinomio xn _ax n- 1 +ax-l

es divisible entre (x - 1)2?

265. Al dividir el polinomio p(x) entre x - a, el resto es A, al dividirlo entre x - b, el resto es B, y si se divide entre x - c, el resto es C. Hallar el polinomio que se obtiene en el resto de la divisi6n de p(x) entre (x - a) (x - b) (x - c), admitiendo que entre los numeros a, bye no hay iguales. Metodo de inducci6n matematica En los problemas propuestos a continuaci6n es conveniente usar el metodo de inducci6n matematica completa. Para demostrar

41

que cierta afirmaci6n es justa para cualquier numero real n, es suficiente que: a) esta afirmaci6n esjusta para n = 1; b) si esta afirmaci6n es justa para cualquier numero real n, sera tambien justa para el numero consecuente n+ 1.

266. Demostrar que 1+ 3 -r' 6 + 10 + ... + (n -I2 )n + n(n+l) 2

= n(n+I)(n+2) 6'

267. Demostrar que 2 _ n(n+I)(2n+l) 12 +-')2,,2 +-' + ... +n 6 .

268. Demostrar que 1 1·23

1

n(n+3)

1

+ 2·3-4 + ... + n(n+l)(n+2) = 4\n+I)(n+2)'

269. Demostrar la f6rmula de Moivre (cos

MN

2

(aqufAD, BCy A1N son las longitudes de los segmentos correspondientes).

68

PROBLEMAS DE MATEMAnCAS ELEMENTALES

521. Demostrar que cualquier angulo plano de un cingulo tetraedro arbitrario es menor que la suma de los otros tres angulos pianos. 522. Demostrar que cualquier angulo tetraedro convexo puede ser cortado por un plano de manera tal, que en la seccion se obtenga un paralelogramo. 523. Demostrar que si en una piramide triangulartodas las caras son equidimensionales. entonces estas son iguales.

3. Lugar geometrico de los puntos 524. Hallar ellugar geometrico de las proyecciones de un punto dado en el espacio sobre un plano que pasa por otro punto dado. 525. Hallar ellugar geometrico de los centros de las secciones de una esfera por pianos que pasan por una recta dada l. Examinar los casos en que la recta cOrta ala esfera, es tangente a ella 0 no tiene con ella puntos comunes. 526. Hallar ellugar geollletrico de los centros de las secciones de una esfera por pianos qu~~ pasan por un punto dado C. Examinar los casos en que el punto C se encuentra fuera de la esfera, en su superficie 0 dentro de ella. 527. Hallar ellugar geometrico de los puntos desde los cuales se pueden trazar ala esfera dada de radio R, tres tangentes que formen con tres angulos pianos rectos un angulo tetraedrico.

528. Hallar ellugar geometrico de los pies de las perpendiculares bajadas desde un punto dado del espacio a las rectas que se encuentran en un plano dado y que se cruzan en un punto. 529. Se tienen un plano P y dos puntos A y B fuera de este plano. A traves de A y B se trazan todas las esferas posibles que hagan contacto con el plano P. Hallar ellugar geometrico de los puntos de contacto. 530. Un plano corta al angulo triedrico par el triangulo ABC. Hallar el lugar geometrico de los centros de gravedad de los triangulos ABC con la condicion de que: a) los vertices A y B son fijos; b) el vertice A es fijo .

4. Valores maximos y minimos 531. Un cuba se corta par un plano que pasa par una de sus diagonales. l,COIllO debera ser trazado este plano para que el area de la seccion sea minima? 532. En una piramide triangular se hacen secciones paralelas a dos de sus aristas que no se cruzan. Hallar la seccion de mayor area.

69

TRlGONOMETRiA

x+v x-y cosx + cosy = 2cos 2 cosT'

TRIGONOMETRIA

x+y

x-y

cosx -cos y = 2sen-2- sen T Observaciones preliminares Expongamos algunas de las formulas que se encuentran en los problemas propuestos a continuacion. 1. Funciones trigonometricas de la suma y la diferencia de dos angulos:

sen (x + y) = sen x cos y sen(x - y) = sen xcos y cos(x + y) = cos x cos y cos(x - y) = cos x cos y

+ cos x sen y. - cos xsen y. - sen x sen y. + sen ~ sen y.

(1) (2) (3) (4)

'

(11 )

(12)

4. Multiplicacion de funciones trigonometricas

1

sen x sen y = icos(x - y) - cos(x + y)i '

(13)

cosxcosy=ticos(x- y) + cos(x + y)i ,

(14)

senxcosy = tlsen(x- y)+sen(x+ y)i,

(15)

~

l-cos2x = 2'

(16)

~

l - cos 2x X = - 2'

(17)

sen- x cos-

5. Expresion de sen x, cos x Y tg x en funcion de

tg i .

2. Angulos doble y triple: 2tg ~

sen 2x = 2sen x cos x, ~ 2 cos 2x = cos~ x - sen x, sen 3x = 3sen x - 4sen 3 x, cos3x = 4cos 3 x-3cosx.

(5) (6) (7) (8)

senx =- -;l+ tg

cos X =

x+~

sen x + sen y = 2sen 2

x+y

x-y

cos-2-, x-y

sen x - seny = 2cosTsen - 2- '

(9)

(10)

(18)

2 ,

(19)

l - tg 2 ~ -

-0-

l+tg"t

tgx 3. Suma y diferencia de funciones trigonometricas:

,

1

2tg~

=--;-. l-tg 1

(20)

6. Funciones trigonometricas inversas.

a) Valores principales de las funciones trigonometricas inversas:

70

PROBLEMAS DE MATEMAnCAS ELEMENTALES

y

= arcsen X , si X = sen y y - ~ :

xy == 6 + 7z. A continuaci6n, obtenemos que

(7 + z y == x 3 + /

+ 3xy(x + y)

x+4y-3z==0.J

De estas ecuaciones se desprende que y==2x, z==3x. Colocando de aquf el valor de y y z en la primera ecuaci6n del sistema inicial, hallaremos quex2==1. Como resultado obtenemos:

ALGEswResoluciones y Soluciones

=O,}

Realizando la verificaci6n nos convencemos de que los dos conjuntos de numeros satisfacen tarnbien al sistema inicial. 57. Fijandonos en que la diferencia de dos ecuaciones del sistema propuesto se descompone en factores, formamos la diferencia entre la primera y la segunda ecuaciones yentre la primera y la tercera. Las dos ecuaciones obtenidas de esta manera,junto con la tercera ecuaci6n del sistema inicial, compondran el siguiente sistema: (u - w)(u + w-l)= (v-w)(v+w-I)=O, (1) w2 +u 2 +v=2.

O,}

Es evidente, que cualquiera soluci6n del sistema inicial satisface al sistema (1). Puesto que, y al contrario, todas las ecuaciones del sistema inicial pueden ser obtenidas sumando y restando las ecuaciones del sistema (1), entonces, toda soluci6n del sistema (1) es al mismo tiempo la soluci6n del sistema inicial y, por consiguiente, estos sistemas son equivalentes. El sistema (1) se descompone en los cuatro sistemas siguientes:

~

2

w- +u +v

= 2,

w2

+

u2

+ V = 2,

O,}

u+w-l = v+w=O,

(4)

w-" + u~~ + v = 2,

..,

?

w- +u-

(3)

(5)

+ v = 2.

En virtud de 10 dicho, es evidente que todas las soluciones de estos cuatro sistemas, y solamente elIas, son al mismo tiempo las soluciones del sistema inicial. Cada uno de estos cuatro sistemas dados puede reducirse sin dificultad a una ecuaci6n cuadrada y tiene dos soluciones. Expongamos las soluciones correspondientes (u, v, w) omitiendo los ca1culos. Soluciones del sistema (2):

-1+-Ji7 ( -I+-Ji7 4' 4

-1+-Ji7J

-1--Ji7 ( -I--Ji7 4' 4

-1-\117 1

4

4

Soluciones del sistema (3): (

(1,0,1),

1

l-2'

(0, I, 1),

(%'

'

r.;:::,.\

2'

-~).

2'

-~).

3

Soluciones del sistema (4):

O,}

u-w = v+ W= 0,

u-w=O} v-w = 0: (2)

u+ w-l v - w = 0,

107

J

108

PROBLEMAS DE MATEMATICAS ELEMENTALES

Soluciones del sistema (5):

(1,1,0),

De {I) se desprende que los miembros derechos de la primera y tercera ecuaciones del sistema (2) son iguales, es decir, que

(-l'

1- 3xy = 7 - 3yz, de don de

As! pues, el sistema inicial tiene en total ocho soluciones.

2 Z-X=-.

Y

58. Restando de la segunda ecuaci6n la primera, obtenemos:

(3)

Puesto que de acuerdo con (1)

z+x=2y,

(4)

entonces, resolviendo conjuntamente (3) y (4) hallamos:

dedonde

(z -

y)(x + y + z) = 3.

Restando de la tercera ecuaci6n la segunda, amilogamente hallamos

1 1 X=Y--, z = Y+ - . Y Y Colocando la expresi6n obtenida para x en la primera ecuaci6n del sistema inicial tendremos que

(y-x)(x+ y+z)=3.

3/-4/ + 1 = 0, dedonde

De las dos ultimas ecuaciones se deduce que

Yl,2=±1, z - y = y -x.

(1)

1 Y34=±-r=- ' , --J 3

Como resultado, hallamos cuatro conjuntos de l1umeros A continuaci6n, escribimos el sistema inicial en la forma siguiente:

(x- y)2 =1-3xy ,1

= 4 - 3xz, ( (y - z)2 = 7 - 3 yz.J (x - z /

(0,1,2) (0, -1, -2);

(~, ~, -~} -

(2)

(-,~, }j'~)'

ALGEBRA!Resoluciones y Soluciones

109

Mediante la verificaci6n nos convencemos de que tados estos coftiuntos satisfacen al sistema inicial.

Este sistema es resoluble unicamente con una condici6n complementaria

59. Multiplicando los primeros y segundo miembros de las ecuaciones entre sf, obtenemos:

(1)

dedonde (1)

escribimos la k-esima ecuaci6n del sistema en la forma

Con esta condici6n, es evidente que se obtendn'i una cantidad infinita de soluciones. A continuaci6n, podemos suponer que a,.: l. (2) Sumando todas las ecuaciones del sistema y haciendo, para simplificar, x + Y + z = t, obtenemos:

De aqui, en virtud de (I), tenemos:

Por medio de la comprobaci6n nos convencemos de que este conjunto de numeros satisface al sistema inicial. Asf pues, el problema tiene una sola soluci6n.

Puesto que segun la condici6n del problema el miembro derecho es positivo, para a = -2 el sistema no tiene soluci6n. Considerando que (3) a,.:-2. hallamos: Ie +/2 + m 2 (4) t=±~ a+2

60. Notemos al principio que siendo a = I el sistema toma la forma

Transformando a continuaci6n las ecuaciones del sistema en laforma:

xk

I n-~ = iJ

.. an (k = 1, 2, ... , )

n .

ak

110

PROBLEMAS DE MATEMATICAS ELEMENTALES

de acuerdo con (4), hallamos de aqu! dos conjuntos de valores dex ey:

1) Supongamos que sea

xy= 2.

(3)

Eliminando x +y de la primera y tercera ecuaciones del sistema inicial, obtenemos la siguiente ecuaci6n respectode z. Z2

-6z+9 =0.

De aqu! se deduce que z(l)=3. Entonces, la primera ecuaci6n da Realizando la comprobaci6n establecemos que las dos temas de numeros satisfacen al sistema inicial. As! pues, en el caso general (a * 1, a * -2) el sistema tiene dos soluciones diferentes.

61. Elevando la primera ecuaci6n al cuadrado y restando de la relaci6n obtenida la segunda ecuaci6n, hallaremos: xy+yz+zx= 11.

x + Y = 3. Resolviendo esta ecuaci6njunto con la ecuaci6n (3), obtenemos: xiI) x~l)

= 1, = 2,

y}l)

= 2,

y~l)

= 1.

(I) 2) Supongamos ahora que de acuerdo can (2)

En virtud de la tercera ecuaci6n, de aqu! se desprende que

(xyY +3xy-10 = O.

(xyL=-5.

Examinemos dos posibilidades.

(4)

En este caso, de la primera y tercera ecuaciones obtenemos:

Resolviendo esta ecuaci6n, obtenemos:

(xy) I =2,

xy = -5.

Z2 -

(2)

6z + 16 = O.

Las raices de esta ecuaci6n son irreales y, por consiguiente, las investigaciones relacionadas con la condici6n (4) pueden no Ilevarse a cabo.

ALGEBRAiResoluciones y Soluciones

ASl pues, solueiones reales pueden ser solamente las siguientes temas de numeros (x,y, z):

(l,2,3)y(2,1,3). Mediante la verifieaei6n nos conveneemos de que ambas temas de numeros satisfacen al sistema inieial. ASl pues, han sido halladas todas las solueiones reales del sistema.

62. Se ve faciimente que los primeros miembros de las eeuaciones pueden ser deseompuestos en faetores, como resultado de 10 eual el sistema toma la forma

(1)

x+z=v,

y+z=w.

Entonees

uv=a,} uw=b, vw=c. Multiplicando todas las ecuaciones entre sl, hallaremos:

(UVW)2 = abc,

(3)

uvw = ± -Jabc.

Ahora, la determinaci6n de todas las solueiones del sistema (2) no representa ninguna dificultad. Eligiendo en la f6rmula (3), al principio, el signo mas y, a continuaei6n ei menos, establecemos que el sistema (2) tiene dos soluciont>s:

(4) y

=

-Mc c

v2

--Jabc =--b- '

w2

=

-Mc a

(5)

Queda resolver dos sistemas de eeuaciones, obtenidos al colocar los val ores (4) Y (5) en los miembros dereehos de las ecuaciones

x+ y =u,} x+z = v, y+z = w.

Hagamos para simplifiear

x+ y=u,

dedonde

u2

(x + y)(x + z) = a,} (x+ y)(y+z)=b, (x + z)(y + z) = c.

111

(6)

Sumando las eeuaeiones (6), obtenemos

(2)

u+v+w x+y+z=-- 2

De aqui, en virtud de (6), se desprende facilmente que

u+v-w u-v+w -u+v+w x=--2- ' y= - -2-' z= 2

(7)

112

PROBLEMAS DE MATEMAnCAS ELEMENTALES

As! pues, el sistema inicial tiene solamente dos soluciones que se determinan por las formulas (7}colocando en estas los valores (4)y(5).

de donde db 2=0, 10 que contradice a la condicion del problema. As! pues, a131"* O. Por esta razon, el sistema (2) coincide exactamente con el sistema (2) del problema anterior. Por consiguiente, este sistema tienedos soluciones:

63. Sumando todas las ecuaciones hallaremos:

la131

_-II_ - , XI -

a 2 + b2 + c 2 xy+xz+ yz= - - - -2

1 (1)

x, =

En virtud de las ecuaciones del sistema, ahora obtenemos facilmente que

-

--fa.I3Y , 1

Yl

~a131

=-13-' Y2

=

~

Z 1 --

l/a131 ., a

-~ 13 '

Es facil de comprobar que estos dos conjuntos de numeros satisfacen tambien al sistema inicial. De este modo, las soluciones (3) y (4) contienen todas I..''> soluciones del sistema. 64. Hagamos

(2)

xy + xz + yz = (3.

(1)

Entonces, el sistema se puede escribir en la forma Aquf hemos introducido, para mayor comodidad, designaciones simplificadas para los quebrados obtenidos. Notemos que si el sistema inicial tiene solucion, entonces, en nuestras condiciones los tres numeros a, 13 y 1 se diferencian de cero. En efecto, supongamos, por ejemplo, que a = O. Entonces 131 = xyz2 = O. Sumando la primera ecuacion del sistema (2) con la segunda y la tercera, obtenemos:

(2)

Sumando todas las ecuaciones de este sistema, hallaremos que

(3)

ALGEBRAIResoluciones y Soluciones

Restando consecutivamente de esta ecuaci6n las ecuaciones del sistema (2), obtenemos:

x 3 = (b + c - a) t 3 ,

y3 = (c + a - b) t 3,

z3 = (a + b - c) t 3 .

Dedonde

x=Vb+c-a·t,

y=Vc+a-b·t,

u = 0, v = 0, w = 0.

113

(2)

Prestamos ademas atenci6n a que si u = 0, entonces, de la primera ecuaci6n de (1) se desprende que v = y de la tercera que w = 0. Por esta rawn, nos limitaremos a examinar los casos enque

°

0.

UVW7:

z=Va+b-c·t.(4)

Colocando estas expresiones en la ecuaci6n (1), hallaremos que, a II = 0, 0 bien

'2 == VU;+c - a)(c +a-b) +V(b +c -a)(a+b -c) + + V(c + a- b )(a + b -

c)

Colocando estos valores de t en las f6rmulas (4), hallaremos dos soluciones del sistema inicial. 65. Hagamos

Este sistema tiene la misma forma que el sistema (6) en la resoluci6n del problema 62. Empleando el mismo procedimiento, obtenemos:

a+b-c x+y=u,

x+z=v,

y+z=w.

u

Entonces, el sistema se escribe de la manera siguiente:

v u + v = auv,} u+w=buw, v+w=cvw. Es evidente, que el sistema(l)tiene la siguiente soluci6n:

2

a-b+c 2

(3)

_~ = - a + b + c .J (1)

w

2

De aqui se desprende que el sistema (1) puede tener una soluci6n distinta de la soluci6n (2) solamente con una condici6n complementaria, a saber:

114 PROBLEMAS DE MAiEMATIC AS ELEMENTALES

a + b - c =a

=t-

0,

O,}

a - b + c = [3 =t- a + b + c = y =t- 0.

(4)

Si se cumple la condici6n (4), entonces, de las f6nnulas (3) deducimos que

2

U= -

2

V=-

W =

~'

a'

2 -. y

(5)

Para concluir la resoluci6n del problema nos queda resolver dos sistemas: 2

x+y=-, a

O,}

x+U = x+z = 0, (7) Y + z

Asi pues, eI sistema inicial tiene solamente una soluci6n nula: x = y = z = 0, y si se cumple lacondici6ncomplementaria(4)tiene otra soluci6n mas que se detennina por las f6nnulas (8) y (4). 66. Por la forma de la segunda ecuaci6n deducimos que x =t- 0, Y =t- 0, Y z =t- 0. Reduciendo el primer miembro de la segunda ecuaci6n a un comun denominador, en virtud de la tercera ecuaci6n,obtenemos: xyz = 27. (1) Multiplicando a continuaci6n la tercera ecuaci6n por z, tomando en consideraci6n (1), tendremos:

2 x+z = --

= 0,

27 + (x + y)z2

~'

(6)

Colocando aqui x + y 9 ( 0 (siendo x = 0 el segundo miembro de la ecuaci6n inicial pierde el sentido), entonces, la ecuaci6n (I) es equivalente a la ecuaci6n

Deaqui

2x + 3 ~ x 2

81. Multiplicando ambos miembros de la ecuaci6n por -J x + I, hacemos x2 + 8x = t. Entonces, obtendremos la ecuaci6n

Esta ecuaci6n tiene una sola ra!z t = 9. Resolviendo a continuaci6n la ecuaci6n x2 + 8x - 9 = 0, hallaremos que XI = -9, x, = I . En virtud de la condici6n adoptada respecto a los val ores de las rakes, la ecuaci6n inicial queda satisfecha solamente siendo x = 1.

1( 'Jj x-I + 'Jj x +

1) = 2x3 .

(I)

yen raz6n de la ecuaci6n inicial 3~

"Ir-

2x + 3 'V x-I x :?j 2

= 2x3 .

(2)

Despues de simples transformaciones obtenemos:

2Fx-l=2~. Elevando ambos miembros al cuadrado nos convencemos de que esta ecuaci6n tiene una sola raiz x = ~~, que satisface tambien a la ecuaci6n iniciaL

-

x

~J x 2 -

1 (3 Vi

- 2 ~p ~1J2) = O.

De aqu! hallamos todos los numeros que pueden servir de rakes de la ecuaci6n inicial. Tenemos directamente:

Resolviendo a continuaci6n la ecuaci6n

hallaremos:

124

PROBLEMAS DE MATEMATICAS ELEMENTALES

Puesto que se buscan solamente las rakes reales, entonces, por consiguiente,

r-

x 2 = 1+ 3.fj. 2

OeaqUl' x4 = 1/ 1+ -3 -fj 2- ,

J

3 -fj Xs =- ~ 1+ --2-'

Es facil comprobar, por medio de la sustitucion, que xl' X 2 y son rakes de la ecuacion inicial. La verificacion directa de los val ores de x 4 y Xs causa ciertas dificultades. Por eso, procedemos de la siguiente manera. Hagamos

X,

En virtud de (4), el segundo miembra de (5) es iguaJ a cera; sin embargo, es facil de ver que a> 0, b > 0, c > 0, y por 10 tanto, la expresion entre Haves es positiva. As! pues, la igualdad (3) queda demostrada. En anaioga forma se establece que Xs es tambien una raiz de laecuacion inicial.

-rx.

al primer miembro y elevando ambos miembros 83. Pasando de la ecuacion al cuadrado, obtenemos que

J

y c

r-

-J X

,.--

. --J X -

-

--

4a + 16 = X - 2a.

Elevando, acontinuacion, ambos miembros de estaecuacion 2 al cuadrado hallaremos que x = G4 es la unica raiz posible de la ecuacion. Colocandola en la ecuacion obtenemos que

= ~2X4

~a2 - 16a+64 = 2~a2 -8a+ 16 -j;2.

y demostremos que

a+b=c. (3) Puesto que x 4 satisface a la ecuacion (2), tendremos que

0,

en virtud de que los radicales son positivos, (I)

(4) y deberemos demostrar que de (4) se desprende (3). N otemos que si en la ecuacion (4), en vez de c ponemos a + b, obtendremos una identidad. Por consiguiente, par el teorema de Bezout, el palinomio c 3 _ 3abc _ a 3 _ b 3 examinado respecto aces multiplo del binomio c «(a+b). Efectuando la division tendremos que

c3

_

3abc _ a 3

_

4 ::; a < 8, la condicion (I) no se cumple, puesto que

8 - a 7: 2(a - 4) - a. Siendo 0::; a < 4, la condicion (1) adquiere la forma

b3 =

= [c - (a + b)]{ c2 + c(a + b) + a 2 - ab + b 2 } •

Siendo a ~ 8 la igualdad (1) se cumple. Por consi~iente, con 2 esta condicion, la rafz de la ecuacion inicial es x = (~ . Siendo

(5)

8-a=2(4-a) - a

ALGEBRAIResoluciones y Soluciones

y se cumple solamente siendo a = O. Por fin, siendo a < 0 la condici6n (1) se transforma en la identidad 8 - a = 2(4 - a)+ Q. Por consiguiente, para

Q

~8

las soluciones de estos sistemas. Resolviendo el sistema (1 ), hallam os:

Y Q::; 0 la ecuaci6n tiene la (mica

(x + y) 2=.l_ 2xy =.l_.l =o.

raiz

5

x=-. a2

125

5

5

Por consiguiente, x + y = 0, y obtenemos dos soluciones del sistema (1):

4

Para 0 < a < 8 la ecuaci6n no tiene rakes.

x - -t-

I-Fa'

84. Elevemos ambos miembros de la primera ecuaci6n y coloquemos en la ecuaci6n obtenida x 2+y de la segunda ecuaci6n. Como resultado tendremos: J

36xy -1 = ~ -

I; + 64xy + 256(xy)

2

Transformando la primera ecuaci6n del sistema (2) a la forma este ultimo se reduce a los dos sistemas siguientes: 3

Elevando de nuevo ambos miembros de la ecuaci6n al cuadrado, obtendremos una ecuaci6n cuadnitica respecto at = x+

.ry:

650t 2 - 85t + 2 = O.

x2 + (1)

i

=i,1 ~ :5 J

= - Fa'

(x + y) 2 = 15

.

Resolviendo esta ecuaci6n hallaremos que tl = I~' t2 Ahora, examinemos dos sistemas de ecuaciones:

t

x2

= ;5 .

")

y=-~'J xy=65,

x+y~- ~,) (2')

xy=-

(2")

65,

EI sistema (2') tiene dos soluciones:

+4xy xy

(2)

Por 10 visto, todas las soluciones del sistema inicial entran en

EI sistema (2' ') tambien tiene dos soluciones:

2 x - - -

5- -/ 65 '

2

Y6

= - -165'

126

PROBLEMAS DE MATEMATICAS ELEMENTALES

Como es faeil de eomprobar, solamente los eonjuntos de numeros primero, segundo, tereero y sexto satisfaeen al sistema inieial. Por 10 tanto, el sistema inieial tiene solo euatro solueiones.

De aqui se desprende que t = 2 (omitimos la segunda raiz - ~ ). Resolviendo el sistema IX

f,--

IJ

85. Hagamos

Entonees, el sistema dado se eseribe en la forma siguiente:

u -v

3

7( 2

=2 u

l'-UV

U

2)

Transformaremos la primera eeuaei6n a la forma

XI

= 4,

/y+l _ 0 - t> . 'V x- Y I

2

Entonees, la primera eeuaeion adquiere la forma

Resolviendo esta ultima ecuaeion junto con la segunda ecuaei6n del sistema hallaremos que U I - 6, VI = 3; u2 = -3, v 2 = -6. Volviendo al sistema inicial obtendremos sus dos solueiones:

x2=-27,

86. Por medio de la sustituei6n

-H =

= 1;

87. Hagamos

uv = 18.

x1=216, Yl=27;

YI

que son tambien las solueiones del sistema inieial. Asi pues, el sistema inieial tiene dos solueiones.

(U - v f + 3uv = '2 uv. Deaqui

}

halJaremos sus dos solueiones:

1 '~

= 3.J

-v

'

x+ XY+: = 9.

v; =u, VY =v. 3

=2

12 -

3t + 2 = o.

De donde II = I, t2 = 2. Examinemos ahora dos sistemas de eeuaeiones:

Y2= - 216.

ry + 1

t 2 0 transformemos la

-IJix-y

primera eeuaeion a la forma X

+ xy + y

I

=1 , f =

7,)

(I)

(2)

ALGEBRAiResoluciones y Soluciones

EI sistema (1) tiene dos soluciones: (- 5, -'-:'3),

x 2 _ y2 xy

(3,1).

=

9}

= 15~

127

(2)

EI sistema (I) tiene dos soluciones reales: EI sistema (2) tiene tambien dos soluciones: Xl

=5,

Yl

=3;

x2 =-5,

Y2 =-3.

EI sistema (2) tiene tambien dos soluciones reales:

_!.J98l-9.

Por consiguiente, el sistema inicial tiene cuatro soluciones

Y3 - ~

!'\}981-9 Y4 = -~ 2

J Ix+y - - =, -1 -y~2 x- - y Vx - y lx- yi

Sin embargo, no es dificil comprobar que s610 dos de los pares de numeros hallados satisfacen al sistema inicial, a saber: ( ,. (5 3) '_ /-v981 + 9

y multiplicando la primera ecuaci6n por x»y, obtendremos: x2 -

i - ~ x 2 - i - 12 = 0 siendo

2 2 Jr--2- 2 x - y + , x - y -12

=0

'

'~

88. Tomando en consideraci6n que

y

2

, ,l

x- y >0

\V

2

_ '

1~981-91

~

2

/

As) pues, el sistema inicial tiene dos soluciones reales.

siendo x - y < O.

Deaquf

89. Hagamos ! 2 -~ x

En relaci6n con esto examinemos dos sistemas de ecuaciones: x2 -

i xy

16,1

= = 15,J

(1)

-12y + I

= t.

Entonces, la primera ecuaci6n se puede escribir en la forma

128

PROBLEMAS DE MATEMAnCAS ELEMENTALES

De aquf t 1,2 = 4 Y obtenemos

x2

Este sistema tienedos solueiones (5, 3, ~), que, como es f£leil de eomprobar, satisfaeen tambien al sistema inieial.

i), (-

(1)

-12y=15.

*

Notando en adelante que y 0, multipliquemos la segunda ecuaci6n por ~~ , como resultado adquirini la forma

90. Liberando la primera eeuaei6n de la irraeionalidad en los denominadores,obtendremos:

-2(~J ~ +(1 +3y 4XJ = 0. ( ~J2 2y 2y f'J"Y Deaqui

Deaqui

5

(2)

4

y

(~J

\y

5 2

4

Despues de elevarla al cuadrado obtenemos la ecuaci6n

3 ( X12 -16

yj

(xJ Y - 12 = 0,

de la que hallamos que

Hagamos en la segunda eeuaci6n I 2

Ii x

+ xy + 4

= t,

(1)

despues de 10 eual se puede eseribir en la forma siguiente

(~l =

2 3

El segundo valor, por 10 visto, no satisface ala eeuaei6n (2), por 10 tanto, podemos limitamos al examen del sistema

x 2 -12y =

I5,}

~=6. y

(2

De aquf I,

= 7,

12

= -8.

+ (_ 56 = O. Ya que en (1)

(z 0,

omitimos la

segunda raiz. Como resultado obtenemos los dos sistemas de eeuaciones siguientes:

x=45 Y'} x 2 + xy - 45

= 0;

(2)

ALGEBRAIResoluciones y Soluciones

92. Hagamos

~x2-6y+l =t;::::O.

x=-"4 y,

5 }

X2

(3)

+ xy - 45 = 0.

Entonces, la primera ecuaci6n se escribira en la fomm

Las soluciones del sistema (2) son: (5,4), {- 5, - 4) Las del sistema (3) son: (15,-12), (-15,12). Las cuatro soluciones satisfacen tambien al sistema inicial.

91. Expresando x por su valor de la segunda ecuaci6n y colocandolo en la primera, obtendremos 2 /1 2 4 1 2 2y + 5 y + 3y --y-- =---+5. IJ 3 3 3 3

(2 -8t+16=0. De aqui t = 4 Ytenemos:

x 2 - 6y -15 = 0. Si ahora hacemos en la segunda ecuaci6n x 2y en consideraci6n (1), obtendremos la ecuaci6n

= t;:::: 0, obtenemos laecuaci6n

(2+ 3t - 18 =0.

(1)

= U Y se toma

9u 2 -241u-13230 = 0, de don de

Haciendo aquf j9y2-34Y-l

129

Uj

= 54, u2 = _ 2~5 .

Obtenemos dos sistemas de ecuaciones

x 2 -6y-15 =0,1

Deaqui

x 2Y=54,J

(2)

(3)

Puesto que segun la condici6n t no es negativa, tenemos solamente una ecuaci6n

Eliminando en el sistema (2)x 2 , obtendremos la ecuaci6n

9y 2 -4y-28 = 0.

2/+5y-18=0,

Resolviendo esta ecuaci6njunto con la segunda ecuaci6n del sistema inicial, hallaremos sus dos soluciones: 17 14

de donde Yj = 2, Y2 = La segunda raiz se omite, puesto que, en virtud de la ecuaci6n x 2 y = 54, conduce a valores de x irreales. Por consiguiente, el sistema (2) tiene dos soluciones reales:

x1=3,

y,=2;

x2=27'

Y2=--i·

-41·

130

PROBLEMAS DE MATEMATICAS ElEMENTAlES

Multipliquemos a continuacion la primera ecuacion del sistema (2) por v y la segunda por u y sumemoslas, como resultado obtendremos:

El sistema (3) se reduce a la ecuacion

u 4 - v 4 + 2u 2v 2

54i +135y+245 = 0, que no tiene soluciones reales. Asf pues, el sistema inicial tiene dos soluciones reales.

93. Hagamos

= -7 uv. 2

En virtud de (4) tenemos:

4(uv)2 -7uv+ 3 = 0.

(5)

De aquf

-rx = u ~ 0; fY =v~O.

(1)

Examinemos ahora dos sistemas de ecuaciones:

Entonces el sistema se escribini de la siguiente manera:

(u 2 -

v2 ) v

= ~ ,1

(6)

(7)

(2)

(u + v )u = 3v-I 2

(UV)l = 1,

2

El sistema (2) tiene la solucion evidente

u= 0;

v

= o.

(3)

°

Considerando a continuacion que u -:;: y, por consiguiente, (en virtud de las ecuaciones) v -:;: 0, multiplicando los primeros y segundo miembros de las ecuaciones (2) entre sf, obtendremos: 4

u -v

4

3 2

(4)

Es evidente que toda solucion del sistema (2) diferente de la (3), se encontrani entre las soluciones de estos sistemas. Multiplicando la segunda ecuacion del sistema (6) por U, en Viliud de la primera ecuacion, hallaremos que if = 2. de aqui, tomando en consideracion (1), obtenemos.

4~ u =--J2,

~

v =2 Analogamente hallamos la soluci6n del sistema (7) que satisface lacondici6n(I): ~3

Vii

U=-- -

2'

V= -

2

ALGEBRA/Resoluciones y Soiw.:iol1t:s

131

Es facil de comprobar que las dos soluciones satisfacen tarnbien al sistema (2). Asi pues, el sistema inicial tiene tres soluciones:

(0, o},

Por medio de la comprobaf!on descu12~imos que el sistema J. inicial tiene solo una solucion ~ a 2 , a 2

,R

94. Elevando ambos miembros de la primera ecuacion al cuadrado,obtendremos:

95. Hagamos

2

I 2 2 a ,jx -y =x--.

2 En virtud de la segunda ecuacion tenemos: I 2

,I x + y

,

2

Fx = U 2

(1)

'V x + y

2

fY = v 2 0.

Y

(1)

Entonces, el sistema adquiere la forma

3a 2 2

=--x.

(2)

(2)

Elevando ahora ambos miembros de la segunda ecuacion del sistema inicial al cuadrado, obtendremos: I 2

°

Este sistema se descompone evidentemente en dos sistemas: U -

4 .Jr--z-z x - y = -a - x 2 .

y

2

u4

+ u 2v 2

+ v4

0,1

v= = b2

,f

u2 + uv + v2 = a,l

De aquf, en virtud de (1) Y(2)

(

u 4 + u 2 v 2 + v4

i

Abriendo los parentesis, hallaremos que x = a 2 . A continuacion, de la ecuacion (1) hallaremos facilmentedos valores dey:

= b2 .J

(2 ')

(2 U)

Resolviendo el sistema (2') hallamos que 3u" = b2 , de donde, teniendo en cuenta (I ), obtenemos:

,(b'w

u=--- -3 '

fb~m v= - - -- . 3

(3)

132

PROBLEMAS DE MATEMAnCAS ELEMENTALES

Pasando al sistema (2"), transformemos ambas eeuaeiones de la forma siguiente:

u 2 + v 2== a _ uv, (u 2 + v 2)2 == b 2 + u 2v 2.

en easo eontrarioel sistema (5), y junto con el tarnbien el sistema (2"), no tiene solueiones que satisfagan la eondiei6n (1). Resolviendo el sistema (5), obtenemos:

De aquf hallamos uv y u2+v2: 2 - b2 uv== -a 2a ' 2

a 2 +b 2

2

u +v =

2a

1 Como resultado tenemos:

r

(4)

'J

2l V

No es dificil demostrar que el sistema de eeuaeiones (4) es equivalente al sistema (2"). De las eeuaeiones (4) obtenemos: 2 l 3a2 -b -I

(u+v\2 =)

2

'I

a 2 ~ (u-v)2 = 3b -a 2a 2

.j

u =_d ) 3a2 _b 2

(5)

Prestemos atenei6n a que el segundo miembro de la primera eeuaei6n del sistema (4), en virtud de (I), debeni ser positivo; tambien debera ser positivo el segundo miembro de la segunda eeuaei6n del sistema (5). De este modo, deberemos suponer eumplida la eondiei6n

(6)

2a

+ hb 2 _a 2 - ~ 2a

1j ,

V~~[ ~3a22:bi +~3b22:a2} Es faeil de ver que, en virtud de la eondiei6n (6), los dos pares de valores (u, v) no son negativos; en efeeto, ya que a 2 ~ b2 , entonees 3a2"b 2 e» 3b 2-a2 . Asf pues, euando se eumple la eondiei6n eomplementaria (6), el sistema inieial tiene tres solueiones:

ALGEBRAlResoluciones y Soluciones

133

As) pues, todos los valores buscados de r se determinan por la desigualdad

r ~ 1. 97. Si hacemos

x y -+-=U y x 2

2

2

Y notamos que'£'- + ~ = U - 2, entonces, la expresion dada se transforma faliime~te en la forma

3u 2 -8u+4.

Si se perturba la condicion (6), solo la primera de elIas.

3. Desigualdades algebraicas 96. Para que el trinomio cuadnitico

sea positivo para todos los valores dex, es necesario y suficiente que a> 0 y que el discriminante D del trinomio sea negativo. En nuestro caso tenemos

= r2 -I > o· D = 4(r _1)2 _4~2 -1)='-8(r -1)< O. a

(1)

(2)

Las desigualdades (1) Y (2) se cumplen simultaneamente cuando r > 1. Sefialemos ademas, que si r = 1 el trinomio examinado en el problemaes identicamente igual a I.

(1)

Si x e y tienen distintos signos, entonces, U < 0 y el trinomio (1) es positivo. Si x e y son de signos iguales, entonces, es faeil verque U ~ 2. Puesto que las rakes del trinomio cuadrado (1) son iguales a 1- y 2, para U ~ 2 el trinomio no es negativo. Asi pues, siendo u < 0 y u ~ 2, el trinomio no es negativo y, por 10 tanto, la expresion inicial no es negativa para cualesquiera valores de x e y reales y no iguales a cero.

98. Notemos que x 2 _ x + 1 > 0 para todos los valores de x, puesto que el discriminante del trinomio cuadrado es igual a - 3 < 0 y el cociente de x 2 es positivo; por esta razon, tenemos derecho a multiplicar ambas desigualdades por el denominador. Como resultado obtendremos:

- 3x 2 + 3x - 3 < x 2 + ax - 2, x 2 + ax - 2 < 2x2 - 2x + 2,

134

PROBLEMAS DE MATEMAnCAS ELEMENTALES

o bien

4x 2 + (a - 3)x + 1 > 0,

(2)

x2-(a+2)x+4>0.

Puesto que siempre ~ a 2b 2e 2d 2 ::::: abed (el signa > para el caso cuando sea abad < 0), entonces, confrontando (1) Y (2) llegamos a la demostraci6n de la desigualdad propuesta.

La primera desigualdad es justa para todos los val ores de x solamente cuando el discriminante del trinomio cuadrado es menor 16 < O. Por razon amiloga que cero, es decir, cuando (a - 3 la segunda desigualdad se cumple con la condici6n de que sea

r-

100. El sistema dado es equivalente al siguiente:

x2+(x+a)2+2x~l,

y=x+a.

La desigualdad Resolviendo conj untamente las dos desigualdades (a - 3)2 -16 < 0 y (a + -16 < 0 respecto aa, obtenemos:

2Y

-4 O. Demostrar que el cumplimiento simultaneo de estas desigualdades es equivalente al cumplimiento de la condici6n puesta en el problema. Admitamos que sea

ALGEBRAIResoluciones y Soluciones

Puesto que q = - p, la expresion anterior se puede escribir enlaforma

K = pa 2 + (I - P)62 _ p(1 _ p};2 + (a 2 _ b2 _ c 2 )p + b2,

Si el triangulo puede ser construido, entonces, las desigualdades (1) se han cumplido y, por consiguiente, D < O. Con esto, en el senti do directo la confirmacion queda demostrada. En el sentido inverso, si D < 0, entonces,

= c 2p2 +

donde a, bye son constantes y p puede tomar cualesquiera valores. As! pues, K representa un trinomio cuadrado con relacion a p. En el caso general, segun sea la magnitud de p, el trinomio K puede tomar valores de diferentes signos. La desigualdad indicada en eI problema es equivalente a que K > 0 para todos los val ores de p. Para esto, como es conocido, es necesario y suficiente que el discriminante del trinomio

(a + b- c )(b + c - a)(c + a - b) > 0.

°

107. Transformemos la parte izquierdade la desigualdad de la manera siguiente: 4 (x + y)(x + z)x (x + y + z) + y2 Z2

2

_b 2

_c

2

r

_4b 2

c= 2

=

=4(x 2 +xy+xz+ YZ){x2 +xy+xz)+ Y"Z2 =

sea negativo (hemos tornado en consideracion que el coeficiente de p2 es igual a c 2 > 0). El discriminante puede ser presentado en la siguiente forma:

(a

(2)

Demostremos que de aqui se desprenden las tres desigualdades (1). En efecto, supongamos que solo uno de los parentesis de la parte izquierda de (2) es positi vo y los dos restantes son negativos, por ejemplo a + b - c < y b + c - a < 0. Sumando estas desigualdades obtenemos que 2b < 0, 10 cual es imposible. De este modo, la confrrmacion queda demostrada en sentido inverso.

(2 b2 D =\a - -c 2)2 -4b 2c 2

D=

137

=4(x 2 +xy+xzy +4yz(x 2 +xy+xz)+ y2 z 2 = =

[2~2 + xy +xz)+ yzy.

La expresion obtenida no es negativa para cualesquierax,yy

z reales, 10 que se exigia demostrar.

= (a 2 _ b2 _ c 2 - 2bc )(a 2 - b2 - c 2 + 2bc)=

= [az _(b+C)2] [az _(b-C)2]= = (a+b +c )(a- b- c )(a+b -c)(a -b +c)= = -(a+b+c )(a +b -c )(b +c -a)(c +a -b).

108. Designando la parte izquierda de la desigualdad 'por z, transformemos z de la siguiente manera: z

= x 2 + 2xy + 3i + 2x + 6 y + 4 = {x + y + 1f + 2(y + If + 1.

138

PROBLEMAS DE MATEMATlCAS ELEMENTALES

Si X e y son reales, los dos prirneros sumandos no son negativos y, por 10 tanto, z ~ 1.

109. Puesto que x = I-;y, entonces, ladesigualdad ademostrar es equivalente a la desigualdad

que se transforma facilmente a la siguiente desigualdad evidente:

100i -40y+4 = (lOy-2)2 ~ O. 110. Yaque d > 0 Y R ~ r > 0, entonces, d 2 R2 - r2 > 0 y

2dr > o.

Por consiguiente, esta desigualdad es equivalente a la desigualdad d 2 + R2 _r2 ~ 2dR. Reduciendola a la forma (d - Rr ~ r2, obtendremos i d - R i ~ r 0 -r ~ d - R ~ r. Porconsiguiente, R- r

~

d

~

R + r.

112. Notemos que la desigualdad dada se reduce a cero cuando b = c, c = a y a = b. Por esta raz6n, por el teorema de Bezout, se divide sin resto por las diferencias a - b, a - c y b - c. Disponiendo los sumandos segun las potencias decrecientes de a y dividiendo entre a - b, obtendremos:

a3(b 2 _c 2 )+ a2(c 3 _b 3 )+b 3c2 _c 3b2 =

=(a - b)[a 2(b 2 _c 2 )+ ac 3 (c -b)+ bc 2(c -b)). Saquemos a continuaci6n de la expresi6n entre corchetes el factor (b - c) y dividamos el polinomio que queda entre a-c. Como resultado obtendremos:

a (b 3

c2

)+ b (c a )+ c (a 3

2 _

2

3

2 _

b2 ) =

=-(b - a)(c -b )(c - a)[ac +bc +ab]. Puesto que por la condici6n a 41 ' I0 que se eXlgla . ,

122. Como es conocido, para cualesquiera numeros reales ay b tiene lugar ladesigualdad

la. bl:-: ; a

2

+b 2 (vease la f6rmula (l) en la pag. 20). 2

Aprovechando a continuaci6n que el valor absoluto de la suma no es mayor que la suma de los valores absolutos de los sumandos correspondientes,obtenemos:

la lb1 + a2 b2 + ... + anbnl :-: ; ia1bll + la2b2 + ... + lanb n 1

~

af + b]2 ai + bi a~ + b~ ++ .. . + 2 2 2 2

=

aJ

2

2

2

2

=

2

+ a2 + .. . + an + bJ + b2 + ... + bn 2

! :-::;;

1+ 1 < - - 2 - =1 .

142

PROBLEMAS DE MATEMATICAS ELEMENTALES

10 que se exigia demostrar.

4. Ecuaciones logaritmicas y exponenciales, identidades y desigualdades

123. Si n = 1, entonces, Xl = I y, por consiguiente, Xl ~ I, as! que la confirmaci6n es justa. Supongamos que es justa para tOOos los val ores de m, donde 1 ~ m ~ n = 1; demostremos su validez para m = n. Si todos los numeros XI' x 2 ' ••. , xn son iguales a la unidad, entonces la confirmaci6n esjusta. Pero si aunque sea uno de estos numeros supera a la unidad, entonces, en virtud de la iaualdad X IX 2 ... x n = 1 existira otro men or que la unidad. t:> Supongamos que la numeraci6n sea tal, que la suposici6n de la inducci6n y la condici6n

XII

< 1,

XI X2·· .Xn-2(Xn-IXn ) = 1 se desprende que

xl + x2 + ... + xn-2 + Xn-lXn ~ n -1, es decir,

Xl + x2 + ... + xn-2 + xn-lx n + 1 ~ n. Puesto que (xn

-1)(1 - Xn-l) > 0,

entonces,

Xn + xn-l - xnxn-l -1 > y par consiguiente, As! pues,

°

X n-i

< 1. De

124. Como se ve de la ecuaci6n, esta tiene sentido solamente para a> 0, a =F-I y b > 0, b =F- I. Para la resoluci6n de la ecuaci6n empleamos la f6rmula de paso a logaritmos con otra base: b loaa _ Cloga t:>-

(vease la f6rmula(2) en la pag. 24). Aqu! c es una base arbitraria (c > 0, C =F- I). La eleeci6n de la base C en este problema es indiferente, s610 haee falta reducirtodos los logaritmos a una misma base. Se puede, por ejemplo, tomar como base eomun a a, por cuanto a > y a =F- 1. Entonces la ecuaci6n se transforma a la forma

°

-alogxal - - og 22 - 2al ogx al og -1

b

al og 2

o despues de la simplificaci6n

(olog2+2 a logb) alogx =3 a log 2 x. De aqui, la primera soluci6n es: °logx

y la confirmaci6n queda demostrada.

Clogb

= 0,

es decir, x = 1.

La segunda soluci6n es:

° logx = 1(a'Og2 +2 a10gb )=laIOg2b2=alOg~J2b2-.

ALGEBRAIResoluciones y Soluciones

es decir,

143

Deaquf

x

=~2b2.

(1_3 10gx)

125. Pasemos a los logaritmos de base 2: haciendo uso de la

1 2 10gx 210gx-4

(1+ 310 g

x)] = 0

y, por consiguiente,

formula (2), pagina 20 obtenemos:

- I- .

[1-

=

210gx-6·

Esta ecuacion es equivalente ala siguiente:

(3 IogX )1 = I,

xI =3;

(3 10gx)2 = 0,

x2 = I; x3

Deaqui {2\ogX)1 =2, (2 10gx

L 3, =

xI =4,

x2 = 8.

1

= - .

9

128. Pasemos en la ecuacion a los logaritmos de base 2. Sobre la base de la formula (2), pag. 24, tendremos

1_210gx ')

2

2

4

- ::---=---+ -log x+ log X = 1.

126. Potenciando con relacion a la base 2, obtenemos

9x - 1 + 7 = 4(3 x - I + 1). Deaqui Por consiguiente,

(3 x - I ) I - "

- J,

XI =2;

127. Pasemos en la ecuacion a los logaritmos de base 3. Sobre la base de la formula (2), pag. 24, tendremos: 1- 3 10gx 31 2 -1 3 + og X - • 1+ logx

1+210gx

Multiplicando ambas partes de la ecuacion por el denominador, pasamos todos los terminos a la parte izquierda y la descomponemos en factores. Como resultado obtenemos: (210gx-l)(210g4x+2 210g3i+ 210g 2 x+2 210gx+l)=0. Parax> 1, el segundo factor, por 10 visto, es positivo y no se reduce a cera. Igualando el primer factor a cera, establecemos que para X > 1, la ecuacion inicial tiene la (mica raizx = 2 .

144

PROBLEMAS DE MATEMATICAS ELEMENTALES

129. Pasemos en la ecuaci6n a los logaritmos de base a(a > 0 y a 1:- I, en el-caso eontrario, 1a expresi6n .; log2x no tendria sentido). En virtud de la f6rmula (2), pag xxxx, obtenemos

°log2x + °log2x = O. 2 ° log a Fx ° log~ ° log ax

°

xl

Siendo log a = b2 . Para a
1. 2) Supongamos que

O 104 , laecuaci6n notiene ralces.

Es facil de ver que toda esta infinita serie de valores de x satisface a la ecuaci6n inicial.

137. Pasemos en la ecuaci6n a los logaritmos de base 2. Como resultado obtendremos la ecuaci6n

x+y=---. x-y

138. De la segunda ecuaci6n hallamos: 2

2 10g a

+ 1=

?

-Iogsen x 2 2 10gsen x

°'

Colocando esta expresi6n de x + yen la primera ecuaci6n, obtendremos:

o bien

Deaquf

2

2

log sen x

2 10g

a

=- - - -

2 Puesto que la magnitud a la izquierda es estrictamente positiva (sen X::F I, de 10 contrario el simbolo sen x log 2 perderia el senti do), entonces, 210g a < y, por consiguiente, para a > 1, la ecuaci6n no tiene rafces. Admitiendo que sea < a < 1, obtenemos:

°

2

logsen x

I

,

210ga

= :t,/i- ~

-

2

- .

10ga

sen x

Pasando a los logaritmos de base 3, transformemos la ultima ecuaci6n a la forma

°

Puesto que 210g sen x < 0, prescindimos del signa mas delante del radical. Entonces I 2

= 2-'J -~

Puesto que 210g3+1::FO,de aquf 31og(x-y)=O x- y

= 1.

Junto con la ecuaci6n (1) esto da el sistema

= 2,1 y = I.]

x+y x-

Resolviendo estas ecuaciones, obtenemos:

y X

= ( - 1)

(I)

k

_J~2Ioga

'arc sen 2 v

2 + nk

3

(k = 0, ± 1, ... ) .

X= -

2'

I

y= -- . 2

y

148

PROBLEMAS DEMATEMAnCAS ELEMENTALES

Por comprobaci6n nos convencemos de que el par de numeros hallado es la soluci6n del sistema inicial

139. Mediante la logaritmaci6n de la primera ecuaci6n respecto a la base c, tendremos: a Clogx

= b Clogy.

(1)

Realizando la potenciaci6n de la primera ecuaci6n, obtenemos = 5 7 , de donde (2)

x . 5Y

Colocando el valor dex de la ecuaci6n (2) en la segunda ecuaci6n del sistema (1), obtenemos laecuaci6n 512+y2 _ 2y = 1, que tiene las rakes

YI ;:::: 4,

De la segunda ecuaci6n haHamos: Clogx-Clogy

Clogx

Y2;:::: 3.

Como resultado obtenemos dos soluciones:

=--. Cl ogy

Colocando aqui Clog y de la ecuaci6n (1), obtendremos: Cl

a C

ogx-t;

logx

b = -;;'

.

bIen

0

cl ogx

I-it b

141. Efectuando la logaritmaci6n de la primera ecuaci6n con relaci6n a la base y, obtenemos la siguiente ecuaci6n cuadnltica respecto a Y logx:

b a

= _

Por potenciaci6n obtenemos: h2

o bien

que tiene las rakes

x = co(b-a).

Yl ogx

Ahora, de la primera ecuaci6n del sistema hallamos: _b_

!!

Si Y log x

y=x b =c b - a . 140. Haciendo uso de la identidad logaritmica a 0 escribamos el sistema en la forma siguiente: 6 10gx + y

xY

10gb

.1

1 2

Ylogx = - .

= 2 , entonces:

= b,

= 7,l

= 5 12

= 2,

(I)

(1) En virtud de la identidad ecuaci6n obtendremos:

a log b

= ~II ,de la segunda oga

Ylog(y -3x )=Ylog4,

ALGEBRAIResoluciones y Soluciones

cuyas rakes son:

dedonde

y-3x = 4.

(2)

Junto con (l), la ecuaci6n obtenida nos da la siguiente ecuaci6n cuadnitica para la determinaci6n de y:

3i-y+4=0. Esta ecuaci6n no tiene rakes reales. Si Y log x =

t, entonces

Y

x2 = 1.

YI=b loga;

x2 = 1,

xI=Glogb La respuesta definitiva es: xI=Glogb,

x

149

Y2 = 1.

143. Pasemos en la primera ecuaci6n a los logaritmos de base x; entonces la ecuaci6n toma la forma

= -JY

e y =x 2. En este caso, en virtud de (2), obtenemos la ecuaci6n

3

(XlOgy+~1 ~J = 10. Xlogy

x 2 -3x-4 = O. Respuesta: x = 4, YY = 16.

142. Realizando la logaritmaci6n de la primera ecuaci6n respecto a la base a, hallaremos:

Pasemos en la segunda ecuaci6n a los logaritmos de base a. Entonces Glogy Glogb 2GI 2 GIogx=--=ogy. Glogb Glog~ De aqui x = ~. Colocando y = ~ en (1), obtenemos la ecuaci6n

Haciendo aqui x log y = t, obtenemos la ecuaci6n

3t 2 - lOt + 3 = 0,



cuyas ralces son: tl = 3 Y t2 = En el primer caso x log y = 3, y = x 3 y, en virtud de la segunda ecuaci6n del sistema inicial, x4 = 81. Puesto que x > 0 e y > 0, en este caso obtenemos una sola soluci6n: xI =3,

YI =27.

Suponiendo a continuaci6n x log y =

mas:

x2=27,

t, hallamos una solucion

Y2=3.

144. Pasemos en cada una de las ecuaciones del sistema a los logaritmos de base 2. Como resultado obtendremos el sistema

150

PROBLEMAS DE MATEMAnCAS ELEMENTALES

1.

(2)

x = y2. (1)

Empleando (2), de la primera ecuacion hallaremos que y = V4. Por consiguiente, 3

x=25, Puesto que x 1=1 (de 10 contrario el primer miembro de la primera ecuacion del sistema inicial no tendria senti do) 2 10 g x 1= 0 Y el sistema (1) se puede escribir en la forma 210gx+ 210gy=210g 12,1

146. Transformemos el sistema, pasando en la primera ecuac ion a los logaritmos de base 2, en la segunda, a los de base 3 y en fa tercenl, a los de base 4. Como resultado obtendremos: 2

210g{x + y) = 3.J

x +Y

4

= 8,

dedonde XI = 6,

YI = 2;

x2 = 2,

Y2 = 6.

x 210gy = Y-JY(1-2 10gx

=3

}1

210gy.j

2

2

14

14

4

\ogz+- logx+- logy= logl6.J 2 2

Despues de la potenciacion, obtenemos el sistema

145. Pasando en cada una de las ecuaciones del sistema a los logaritmos de base 2, obtendremos:

2 2\ogx

"1 12 12 2 logx+- logy+ - logz= log4, I 221

~ 13 13 3 I .Jlogy+_ logz +- logx= log9, r'

Despues de la potenciacion, obtenemos:

xy = 12,

2

y=25.

x~

=4,l

y-Jxz =9,

(1)

De la segunda ecuacion del sistema (1) hallamos Xl = y, de donde

,-- J zjry = 16.

(1)

Multiplicando las ecuaciones del sistema (I) miembro a miembro hallaremos:

ALGEBRAIResoluciones y Soluciones

Puesto que x > 0, y > 0, Z > 0, entonces,

151

148. Dividiendo ambos miembros de la ecuaci6n entre 4\ hallaremos:

xyz = 24.

(2)

Elevando la primera ecuaci6n del sistema (1) al cuadrado y empleando (2), obtendremos: 16 2

1

(3\x -l-(3JX . = - 13 - -. 4 13 \4) 2 1

I

1

Deaquf

X= - = - . 24 3

Amllogamente hallamos que y

= 21;

Yz

= 332 . Por compro-

y, por consiguiente,

baci6n nos convencemos de que los tres numeros hallados son la soluci6n del sistema.

147. Pasando en la primera ecuaci6n a los logaritmos de base 2 y realizando a continuaci6n la potenciaci6n, obtendremos:

i

(1)

-xy=4.

3 X=-. 2

149. Colocando el valor de y de la segunda ecuaci6n en la primera., obtendremos:

De aquf, 0 x = 1,0 bien

1 2 x+ -- =-2x+2 x x2

La ecuaci6n (1), junto con la segunda ecuaci6n del sistema inicial, da el sistema

x2 +

i

i

y, por consiguiente,

= 25,1

-xy =

4.]

1

X=

(2)

VS.

Respuesta: Este sistema tiene dos soluciones que satisfacen a las condicionesy> x,y > 0, a saber:

1

Yl -

-i2'

Y2

= 4.

xl

=Yl =1 ,

150. Haciendo aX forma siguiente:

=u

y aY

= v, escribimos el sistema en la

152

PROBLEMAS DE MATEMATICAS ELEMENTALES

u1 + V 1 = 2b,}

Ul=

vl

De estas dos ecuaciones se desprende:

(u + vf = 2(b + c) , (u - vf = 2(b - c) . Puestoque los valores buscados de U y v deben ser positivos, la primera ecuaci6n se reduce la ecuaci6n

(1) La segunda ecuaci6n demuestra que la solubilidad del sistema requiere, ademas de que sean positivos los numeros by c, el cumplimiento de la desigualdad (2)

b? c.

-fi{-Jb+c--Jb-c), 2

UV=C.

= -fi (-Jb + c + -Jb - c). 2

Hemos hallado dos soluciones del sistema (1), ademas, al cumplir la condicion (2) todos los valores de las incognitas, por 10 visto, son positivos. Las dos soluciones correspondientes al sistema inicial son: xl=Glogu1 ,

Yl=Gl ogv1 ;

x2=Glogu1 ,

Y2=Glogv2'

Ahora podemos confmnar que para que el sistema sea soluble es necesario y suficiente que b > 0, c > 0 y b? c. Al cumplir estas condiciones el sistema tiene dos soluciones. 151. Multiplicando ambas ecuaciones entre sf, obtendremos:

Al mismo tiempo, U -

v = ±-J2(b -

c).

(3)

Resolviendo conjuntamente las ecuaciones (1) Y(2), en el caso del signa mas, obtendremos:

De aqui, en virtud de que x e Y son positivos, se desprende que 0 bien xy = 1,0 xY -:F1, yentonces

x+ y=2n. Ul

= -fi (J b+ c + -J b - c ), 2

vl=

-fi(-Jb+c--Jb-c). 2

En el caso del signa menos, obtenemos:

(I)

Examinemos al principio el segundo caso. La primera ecuacion del sistema inicial toma la forma.x2 n)l', de donde (2)

ALGEsRAi'Resoluciones y Soluciones

Colocando este valor de yen la ecuaci6n (1), obtenemos que

y por consiguiente, en virtud de la primera ecuaci6n

x 2 +x-2n = o.

324

x- y

(3)

El valor correspondiente dey 10 hallamos haciendo uso de (2):

=! (J8n + 1 -

= 2 x - y ·81.

Deaqui 22 = 2x - y , esdecir,

Esta ecuaci6n tiene la unica raa positiva

f

1 (4) 4 En el segundo caso, cuando xy = 1, y = -;-, y la primera ecuaci6n del sistema inicial adquiere la forma Yl

X-

y

3x + y

=2}

De la segunda ecuaci6n hallamos: 324

= 2x - Y (3x+ y)2(X- Y ),

(1)

= 3~

X- y

(2)

3x + y

=2}

= -3~

(3)

Soluci6n del sistema (2): 5

En virtud de que x y n son positivos, esta igualdad puede ser valida solamente en el caso en quex = 1. De este modo hallam os una soluci6n mas: X z = 1'Yz = 1.

= 2.

Resolviendo la ecuaci6n (1 )juntocon la primera ecuaci6n del sistema inicial, obtenemos dos sistemas:

xl

152. Transformamos el sistema a la forma

153

=4'

3

y\ =- - .

4

Soluci6n del sistema (3):

Y2

9

= --. 4

Por comprobaci6n nos convencemos de que los dos pares de numeros satisfacen al sistema inicial. 153. Hagamos ;; = u. Si u = 1, es decir, p = q, entonces, cualquier par de numeros iguales y positivos satisfacen al sistema. Por esta raz6n, consideraremos que U:l: 1. De la segunda ecuaci6n obtenemos que x = yX. Realizando la logaritmaci6n de la primera ecuaci6n y empleando esta igualdad, tendremos:

154

PROBLEMAS DE MATEMAnCAS ELEMENTALES

y log y

(a - yX-I)= 0.

2=Glog(c -b )+Glog(c + b). Deaqui,

Puesto que y > 0, entonces, 0 bien logy =0,0 bien a = yX-I. En el primer caso obtenemos que x] = 1, y] = 1. En el segundo caso,obtendremos: x2

-.!L

= a a-I,

-.L

2=

1

+ - :---c+b log a

c-b log a

y, por consiguiente,

Y2 = a a-I.

Ambos pares de numeros satisfacen tambien al sistema inicial. 154. Efectuando la logaritmacion de am bas ecuaciones, obtendremos el sistema

1 ·obtendremos f:kil156. Utilizando Ja fonnula nlogm = -;;--1 ogn mente que

ylogx = xJogy,l

I· , . I de I que determmamos a re actOn guiente. x=ay.

b2 - k toga =

(1)

xlogp = yJogq.J log q yx = logp =a;

por conSl-

I

n (' . b·-I

toga

toga-a l ' 10gb )20) =

k=O

(2)

2kb

I

11

k=O

(

2kb

1)

toga-kGtogb 2

2

Si p = q, el sistema tiene un numero infmito de soJuciones del tipo x = y =a, donde a es cualquier valor mayor que cero. Si p -j; q, entonces colocando el valor de x de la formula (2) en Ja primera ecuaci6n del sistema (1 ), hallaremos: -.!L

x=aa-I,

_ 1_

y=aa -l

Por consiguiente, para la condici6n p unasoluci6n.

-j;

q el sistema tiene

155. Realizando la logaritmaci6n de las dos partes de la igualdad a2=c2 "b 2 ,obtenemos:

-

- _.._ - -

n

*) EI sfmbolo

I

ak significa la suma ao + at + a2 + ... + ank=O

ALGEBRA/Rcsoluciones y Soluciones

De aqui se desprende que

157. b

a

155

10ga loga b =(a"IOgb) blog h loga =bb log h loga

log

,vI

b

= bloga .

158. Tenemos que _

C -

a}a2 ... an

_ -

a aq ... ~aq .

n-l ) _

n{n-l)

n

- a q

-2-

.

b

og~=

ci

b _ alog~ _ A og - b ( I) . loge n+ n ~- al ogq

Pero

Por eso,

0

loge N log a

__ 1 _ __ 1_ N

=

loga 1

N

NI

1-

NJogb NJogc

b

=

N

log fb

Asi pues b es el valor medio proporcional entre a y c. ReaIizando a continuacion la logaritmacion de la igualdad (2) respecto a cualquier base Ny efectuando los computos en orden inverso, conc1uiremos la demostracion de la afirmacion propuesta.

Pasando aqui a los logaritmos de base N, obtendremos:

NI oga+ N logb+ N logc= N logabe.

,

_ oga. "Iogc

logb

b

I 1 I 1 + + =--alogN b logN clogN abclogN'

2AB 2nB + n(n -I)A'

159. Utilizando la igualdad a 10gb = ~I} ,transformemos la o"a fonnula dada de la manera siguiente: N

(1)

160. Se debe considerar N 1= I, de 10 contrario, el quebrado en la parte derecha de la identidad se hace indetenninado. Dividiendo la identidad a demostrar entre a log N blog N Clog N la sustituimos por la siguiente identidad equivalente:

a logq = b logq = alogb = A b loga qlogb B Cl og b =

C

ogt;'

puesto que el factor : :OgC 1= O. Realizando la potenciacion de la igualdad (1) obtenem~gs~ b C (2)

a

Empleando la formula de pasode un sistema de logaritmos a otro, obtenemos:

NI

N

Jog a .

Puesto que es evidente que la ultima identidad tiene lugar, el problema queda resuelto.

156

PROBLEMAS DE MATEMAnCAS ELEMENTALES

161. Tenemos: Gl ogx

1

0
0, la desigualdad dada es equivalente a la desigualdad

10 que se exigia demostrar. 'd 'd I .. b Cloga 162. Emp Ieand 0 Ia 1 entI ad ogantmlca loga = Clo b' transformemos la parte izquierda de la desigualdad dada ~e la manera siguiente:

Pero a> 1, por eso, realizando la logaritmaci6n de la ultima desigualdad respecto a la base a (esta operaci6n conduce tambien a una desigualdad equivalente), obtendremos: Glog2 x> 2. De aqui hallaremos defmitivamente: o bien G log x > .f2, y, por consiguiente, x > a J2 ; o bien G log x < -.f2 yentonces < x < a --Ii .

°

Entonces, la desigualdad dad1!- adquiere la forma:

164. Por el senti do del problema x > 0, por eso, la desigualdad dada es equivalente a la desigualdad

_ :IOgx > 1. 210g2

t

Pu~sto que 2 > 1 y > 1 Y por la propiedad de los logaritmos ~ log 2 > 0, entonces, la desigualdad anterior es equivalente a la desigualdad

De aqui, observando ademas que por el sentido del problema

x> 0, obtendremos definitivamente:

Glogx(x+l)< Glog (2x+6). Puesto que a> 1, entonces x (x + 1) < 2x + 6,

0

bien

x 2 -x-6 < 0. Resolviendo esta desigualdad cuadnitica con la condici6n de que x > 0, obtenemos que

l) 100B. Puesto que, ademas,p>O, A>O, B>O, entonces la expresion que figura en la parte derecha de la desigualdad (1) tiene senti do. 226. Al final del primer ano en la parcela forestal habia una cantidad de madera igual a a ( 1+ L

\

'I - x = a] ,

100)

y asf sucesivamente. Por fin, al final del enesimo ano la cantidad de madera era

Es necesario determinar x. Haciendo, parasimplificar laescritura, 1+ I~O = k, de la ultima igualdad obtendremos que x = kan_l - aq. Expresemos su valor de la ecuacion anterior. Obtendremos:

Por consiguiente,

al final del segundo ano,

= k 3a n-3 -

2

k x - kx - aq.

Continuando del mismo modo, expresaremos por fin ao mediante a l y obtendremos la siguiente ecuacion respecto ax: -

100 )

al final del tercer ano,

por

Pero,

x

a1(I+L l-x=a2'

0,,_1

x

=

k n a - x (kn-l + k n - 2 + ... + k) - aq.

ALGEBRAIResoluciones y Soluciones

183

Deaqui Deaqui

x=a kn - q (k_I)=a(l+tfot -qL. k -1 (1 + 1 100

n

L)n _

~ 100 227. Antes del transvase la concentracion de alcohol q; era: en el primer recipiente ql = I, en el segundo recipiente q2 -

t'

en el enesimo recipiente qn

= kL 1 •

Supongamos que despues de todos los transvases las concentraciones se hicieron respectiva mente iguales a: PI> P2 , ···, Pw Entonces, PI = 1 y, siendo i> 1, Pi se detemlina por la ecuaci6n

qi + Pi=l

(i=2, ... ,17).

2

Esta ecuaci6n se obtiene dividiendo la cantidad de alcohol v v

qi 2 +Pi-l 2·

que result6 en el i-esimo recipiente despues de llenarlo con el contenido del

(i -l)esimo recipiente, por el volumen total v del

recipiente. De este modo,

P = qn + Pn-l 2 11

= ql1 + qn-l +~ qn-2 + Pn-3 = ~ + ql1-1 + qn-2 + PI1-3 = 2 ...

22

22

+ qn-l +

= qn

2

22

1

2

2

22

23

+3L;- y~ =_l_+ __l _ ~

... 2n- 1

2n- i

2e- 1

22 e- 2

1

... + - n -+ - 11 -. 1 1 2

Para k

-

-::1=

k

2

-

2, la ultima suma es igual a 1

Pn = 2k

kl1 - 1 ')11-1

t --~

2 n - 1 - k n- I

1

_ 1_ _ 1_

+ 2 n - 1 = (2k

228. La fracci6n tiene la fonna

t- (2 - k) + 2 1

... ,

1 n- 1

+1' donde pes un nlunero p -

entero mayor que cero. Las condiciones del problema se escriben en forma de desigualdades

P+2

I

- -> p2 + 1 3'

P -3 1 0 < - -- < p2 _ 4 10

Transfonnemos la primera desigualdad a la fonna

P =Hn+Pn-1 n 2

23

184

PROBLEMAS DE MATEMAnCAS ELEMENTALES

Resolviendo la correspondiente ecuacion cuadrada, obtendremos:

Pl,2 =

3±m 2

7. Problemas diferentes 229. Tenemos:

De la desigualdad 0 > p2 - 3 P - 5 obtenemos P2 < P < Pl. Pero P2 < 0 y P > 0, por eso

O

G2

= Ab2 ,

G3

= Ab3 .

(2)

1· Ademas. en este caso, debe cumplirse la siguiente bI2 + b22 + b32

7:-

0,

(3)

de 10 contrario, los tres numeros sedan ceros y el primer miembro de (1) no dependerfa de x. Supongamos ahora que, al contrario, se han cumplido las condiciones (2) y (3); entonces

P=-XI-X2 =-2ex>0

y q=XI X 2 =ex 2 +p2 >0 .

°

Supongamos que, al contrario, se conoce que p > y q > 0. Entonces, en el caso en que XI y x 2 son reales, de la igualdad XI . x2 = q se desprende quex i y x 2 son de un mismo signa y de la igualdad XI + x 2 = - p se deriva que las raices son negativas. Si XI

= ex + if3, x2 = ex -

=2a,

XI

iP, f3 7:- 0,

entonces,

y por consiguiente, ex es negativa.

XI

+ Xl = - P =

ALGEBRAI'Resoluciones y Soluciones

243. Puesto que las rakes de la ecuaci6n x2 + px + q positivas, el discriminante de la ecuaci6n es

D = p2 -4q ~ 0,

=

°

son

189

Es evidente que la afirmaci6n es justa si se exige que sean simultineamente 1 - pr >

(1)

°y

p - 2rq < 0,

es decir que y los coeficientes satisfacen las desigualdades

p = -xl - x2 < 0. q=xlx2 >0.

(2)

1 r>-, p

(7)

r>L. 2q

(8)

(3)

Supongamos ahora que Y1 e Y z son las rakes de la ecuaci6n (4)

As! pues, para los valores negativos de r que satisfacen las condiciones (7) y (8)'Y 1 e Yz son positivos. Cuando no se cumplen estas condiciones, una 0 ambas rakes de la ecuaci6n (4) no son positivas.

yen virtud de (1) no es negativo cualquiera que sea el valor de r. Por consiguiente, Y1 e Yz son reales para todos los valores de r. Teniendo en cuenta (2) y (3), por las f6rmulas de Viete, para r ~ 0, obtenemos que 1- pr YlY2 = - - > (5)

244. Supongamos al principio que p ::j:. 3. Para que las rakes de una ecuaci6n cuadnitica con coeficientes reales sean tambien reales es necesario y suficiente que el discriminante D de esta ecuaci6n no sea negativo. Tenemos que

q/ +(p-2rq)Y+l- pr = 0. EI discriminante de esta ecuaci6n es igual a

Dl = 4r2q2 + p2 -4q

°

q

y, por 10 tanto'Y l ey z son de un mismo signo.Acontinuaci6n, ~+h=-

p-2rq q

>0

W

y, por consiguiente, para r 20, Y1 e Yz son positivos, con la cual el problema queda res!lelto.

D = 4p2 -24p(P-3)= 4p(18-5p). Por eso D 2

°

cuando

0-:; p -:; 3,6.

(I)

Las rakes r~alesxl y X z seran positivas cuando, y s610 cuando, su suma y su producto sean positivos, es decir, cuando

190

PROBLEMAS DE MATEMATlCAS ELEMENTALES

xl +X2

2p

=- -

p-3

>0,

Xl . X2

6p

=- -

p-3

> O.

(2)

El sistema de desigualdades (l) y (2) se satisface en el caso en que

3 O. Por esta raz6n, todos los valores buscados quedan detenninados por la condici6n

3~ p

~

3,6.

245. Demostremos la afirmaci6n pro oposici6n. Supongamos que sea a O. Entonces, para las raices XI y X 2 tenemos:

"*

- b± ~b2 -4a(c+A)

- b + ~ b 2 - 4a(c + A) > 0

y, por 10 tanto, la rafz

-b+ ~b2-4a(C+A) c y ia»bj< c, entonces, (a+b)2> c 2 y (a-b )2< c2 . Por consiguiente, D < O.

= 0,

=

= 2(Xl +x2 +x3)= 2.

Y2

= 1-

x2 e Y3

= 1-

x3' entonces,

(1- XI )(1 - X2) + (1- x2 )(1 -

X3) +

+ (1- x3)(I- Xl)= 3 - 2(xl + x2 + X3)+ xlx2 + x2x3 + xlx2

=1

192

PROBLEMAS DE MATEMATICAS ELEMENTALES

y, portin,

Para que con los segmentos de longitudesx 1,x2 y X3 sepuede construir un triangu\o es necesario y suticiente que

(2)

La nueva ecuaci6n tiene la forma y3 _

2i + y + 1 = o.

Este hecho fue demostrado en la resoluci6n del problema 106. Para obtener la condici6n planteada en el problema, expresemos la parte izquierda de la desigualdad (2) por medio de p, q y r. Con este tin, valgamonos de la dependencia entre las rakes y los coeticientes de una ecuaci6n

252. Supongamos que sea XI

= p-d,

x2

= p,

x3

= p+d.

Entonces, xI + x2 + x3 = 3 p; por otra parte, por la f6rmula de Viele xI+x2+x3=-a. De aqui 3p=-a y, por consiguiente, a x2 = P = --. 3 Colocando esta raiz en la ecuaci6n, obtenemos:

+ X2 + X3 = -p, XI X2 + XI X3 + X2 X3 = q, XIX2X3 = -r.

Xj

Escribamos ahora la condici6n (2) en la forma siguiente:

deaqui

- p3 _2p2(Xj +x2 +x3)-4p(xjX2 +x\x3 +X2X3)-8xjX2X3 > 0 y, por consiguiente,

de donde

23 1 c=--a +-ab. 27 3

p3 _ 4 pq + 8r > O.

°

253. Supongamos que xl > 0, x2 > 0, x3 > son las raices de la ecuaci6n dada. Ateniendonos a la indicaci6n, estudiemos la expresi6n (I) (XI +x2 -X3)(X2 +x3 -XI)(X3 +X\ -X2)·

254. Sea Xo la raiz comun de las ecuaciones. Colocando Xo en ambas ecuaciones y substrayendo una ecuaci6n de la otra, hallamos:

Xo = 32 - q\ Pl- P2

7=

0.

ALGEBRAIResoluciones y Soluciones

Supongamos que x2 + ax + b es el cociente de la division del trinomio x3 + PIx + qi entre X - xo. Entonces,

x 3 + Plx+ qi

= (x-xo)~2 + ax+ b).

Igualando en esta identidad los coeficientes de x 2 a los terminos independientes, hallaremos: a = Xo y b = uq/xo' De aqui se deriva que las otras dos rakes de la prim era ecuacion se determinan por la formula

Iv+ 1 Xo = - - . Iv

193

(2)

Asi pues, si existe la raiz comun, ella esta enlazada con Iv por la fOrmula (2). Se puede comprobar facilmente que la fraccion

At efectivamente satisface a ambas ecuaciones (por 10 visto, es suficiente establecer este hecho solamente para la segunda ecuacion). As! pues, las dos ecuaciones (I) tienen una raiz comun para cualesquiera valores de Iv :f: 0. Esta raiz se determina por la formula (2). 256. Primera resoluci6n. Supongamos quex" x 2 y X3 son las rakes del polinomio P(x). Por las formulas de Viete tenemos:

y las de la segunda ecuacion, por la fonnula

de donde se desprende facilmente que 222

255. Es facil comprobar que para A= 0, las ecuaciones no tienen raiz comun. Seaxc la raiz comun de las ecuaciones para cierto valor de Iv :f: 0. Entonces,

4g - x~ -

Xo - (Iv + 1) =

a,}

Xl

Puesto que

XI' Xl

+ x2 + x3 + 2P = O. Y X3 son reales y difieren de cero

(q :f: 0), entonces, xf + xi + xi >

°

y, por consiguiente,p < 0.

(1)

Segunda resoluci6n. Se ve facilmente que entre las tres ralces del polinomio P(x) habra dos desiguales. En el caso contrario

Multiplicando la segunda igualdad por Xo y restandola de la primera, hallaremos:

P(x) representaria un cuba exacto: p(x) == {x - xO)3, 10 que,

40 - Xo - (Iv + I) = o.

evidentemente no tiene lugar.

194

PROBLEMAS DE MATEMAT'CAS ELEMENTALES

Supongamos ahora quex, Yx 2' son dos rakes desiguales del polinomio Y seax, 0.

XIX2

(1) (2)

Supongamos al principio que las tres rakes son reales. Entonces, de la condicion (2) se deriva, que, por 10 menos, una de elias es positiva. Si al mismo tiempo resultaran positivas dos rakes, entonces, en virtud de Ia misma formula (2) deducirfamos que la tercera raiz tambien es positiva, 10 que contradice ala condici6n (1). De este modo, para el caso en que las tres raices sean positivas el problema queda resuelto. Admitamos ahora que x, es una raiz irreal de la ecuaci6n, entonces, como es sabido, la eCllaci6n tiene una rafz compleja conjugada x2 + Xl. Puesto que en este caso XIX2 = XI Xl > 0, de la igualdad (2) obtenemos;

= -12.

(4)

De las ecuaciones (1) Y(3) obtenemos: Yl - Y2 =-a

(5)

Yde las eClIaciones (2) Y(4)

1i Y2

3 2

(6)

Resolviendo conjuntamente las ecuaciones (5) Y(6), hallamos:

YI = -3a,

Y: = -2a.

(7)

De este modo, si para ciertos val ores de a Y bIas eClIaciones tienen raices comunes; las terceras raices de estas ecuaciones se

ALGEBRAIResoluciones y Soluciones

determinan por las formulas (7). Colocando y\ = -3a en la primera ecuacion y Y2 = -2a en la segunda, obtenemos:

-18a 3 + 18 = 0

y -18a3 -

2ab + 12 = O.

= 1,

b = 2.

(8)

Colocando estos valores en la ecuacion hallaremos facilmente que

y

x 3 + x 2 + 18 = (x + 3)~2

- 2x + 6)

x 3 + 2x + 12 = (x + 2)~2

- 2x + 6).

Por consiguiente, para los val ores indicados de a y b, las ecuaciones tienen efectivamente dos rakes comunes. Estas rakes se determinan por la formula Xl, 2

Asi pues, la parte izquierda de la igualdad a demostrar satisface ala ecuacion cubica (1) x 3 -6x-40=0. Es facil comprobar que parax=4laecuacion (1) se satisface. Dividiendo la parte izquierda de la igualdad (1) por x «4 obtendremos la siguiente ecuacion para la determinacion de las otras dos ralces:

De aquf se determina el unico par de valores reales a

=-I±-J-S.

259. Designemos la parte izquierda de la igualdad por A. Tenemos:

x 2 + 4 x + 10 = O. Esta ecuacion tiene rakes irreales, puesto que su discriminante D =--24 < O. Por 10 tanto, la ecuacion (I) tiene la unica raiz real x = 4 Y puesto que A es notoriamente un numero real, entonces A = 4, con 10 cual el problema queda resuelto.

260. Es facil ver que la expresion que se exam ina se reduce a cero si dos cualesquiera de los numeros a, bye son iguales entre sf. De aqui, segun el teorema de Bezout, se desprende que debe dividirse sin resto por cada una de las diferencias

(b - c), (c - a), (a - b). Esto sugiere que la expresion dada es el producto de los factores indicados. Efectivamente, tenemos:

A3 = 20 + 14-12 + 3~~0 + 14-J2)i~20 -14J2 +

a 2 (e - b)+ b2 (a -e)+ e 2 (b - a)== a2e _ a2b +

+ 3~)20 + 14--J2

~(20 -14J2 )2

+ b2 a -b 2e + e2b - e2a == a 2 (e - b)- a(e 2 _ b2 )+

= 40 + 3 ~ 400 -

2 . 14 2 A

+ 20 -14-12 =

= 40 + 6A.

195

+be(e-b)=(c-b)[a 2 -ae-ab+be]= = (e -b ) [a{a -e )-h{a-e )]= (e - b)(b -a)(e - a).

(I)

196

PROBLEMAS DE MATEMATICAS ELEMENTALES

Puesto que a, bye son por pares diferentes, la afirmaci6n queda demostrada.

261. Observemos que para x = - y la expresi6n dada se reduce a cero. Por consiguiente, por el teorema de Bezout, se divide sin resto entre x Iy. Para efectuar la divisi6n representemos x + y + z en forma de la sum a de dos sumandos: (x + y) y z. Elevando esta suma al cuba obtenemos:

El primer miembro deesta igualdad se descompone faciimente en factores

a 2(b + c)+ ab(e +b)+ ac(b + c)+ bc(b + c)= = (b + c)(a 2 +ab + ae + be)= (b + c)(a+ b)(a+c). Puesto que el ultimo producto es igual acero, entonces, por

10 menos uno de los facto res es igual a cero, de donde se deduce la resoluci6n del problema.

[(x + y)+z]3 _x 3 -

i

= (x + y)3 + 3(x + y

f z + 3(x + y)z2 _ x3 _ y3 =

_z3 = 263. Sean a y

13 las ralces del trinomio cuadratico x 2 + px + q.

Si el binomio x 4

=3(x+ y)[z2 +z(x+ y)+xy]. El trinomio cuadrado respecto a z, que figura a la derecha entre corchetes, puede ser facilmente descompuesto en factores, puesto que es evidente que sus rakes son «x y «y. Como resultado obtenemos:

entonces, a y

_

1 se divide entre e\ trinomio indicado sin resto,

13 son ralces del binomio. Se ve facilmente que es

justo tambien 10 opuesto: si a y

13 son rakes del binomio x 4 -1,

entonces, este se divide entre x 2 + px + q sin resto *). Las raices del binomio x4 -1 son los numeros, 1, -1, i, -i. Por eso, tiene lugar la descomposici6n

262. Multiplicando ambos miembros de la igualdad dada por abc(a + b + c) la reduciremos a la forma

(ab + be + ac )(a + b + c) - abc

= O.

En virtud de 10 dicho mas arriba los trinomios que nos interesan pueden ser solamente aquel\os que representan un producto de los dos factores que figuran enla parte derecha de (1).

Abriendo los parentesis, obtenemos: *) En cste caso, si del dividendo.

a = 13,

el numero

a

debera ser raiz multiple tambien

ALGEBRAIResoluciones ySoluciones

Componiendo todas las combinaciones posibles hallaremos Ci ( 4)=6 trinomios.

Para que este polinomio se divida por x-I sin resto, en virtud del teorema de Bezout debe cumplirse la igualdad

(x -I )(x + I) = x 2 - 1,

n - a(n -

i) = x 2 - (1 + i}x + i, (x -1)(x + i) = x 2 - (1- i}x - i, (x + 1)(x - i) = x 2 + (1 - i) x - i, (x + 1)(x + i) = x 2 + (1 + i}x + i, (x - i )(x + i) = x 2 + 1. (x -1)(x -

197

2) = o.

Por eso la divisibilidad tienen lugar para cualquier numero entero positivo

n n>2 y a=--. n-2 265. De las condiciones del problema se desprende que

A,}

pea) = pCb) = B,

Con estos por 10 visto se agotan tOOos los trinomios buscados.

pee) = c.

264. Representando el polinomio dado en la forma

(1)

Dividiendo el polinomiop(x) entre (x-a)(x-b)(x-e), representemoslo en la forma

xn - 1- ax(xn- 2 -1), dividamoslo por la diferenciax« 1 valiendonos de la formula

p (x) = (x - a )(x - b )(x - c )q (x) + r (x) x - 1 k - - - =I+x+ ... +x x-I

(2)

k+l

(1) *)

Es evidente que r(x) es un polinomio no superior al segundo orden. Escribiendolo en la forma

Como resultado en el cociente obtendremos el polinomio

x n-l

(n-3 +x n- 4 + ... +x+. 1) +x n-2 + ... +x+ 1- axx

*) La f6rmula (I) se comprueba con facilidad directamente, es mas, ella coincide con la f6rmula de la suma de k lI::rminos de una progresi6n geometrica.

(3) Coloquemos en la identidad (2) sucesivamente x = b, x = c. En virtud de la igualdad (1) obtendremos el siguiente sistema de ecuaciones para la determinacion de los coeficientes I, my n del polinomio (3):

x

= a,

198

PROBLEMAS DE MATEMAnCAS ELEMENTALES

fa 2 + ma + n = A. "1I Ib 2 + mb + n = B, ~ "

(4)

I

le 2 + me + n =

C.j

Resolviendo este sistema hallaremos:

266. Por 10 visto, la formula es justa para n = I. Supongamos que la formula esjusta paracierto numero n; demostremos que entonces ella sera tambienjusta paran +1. Designando por Sn la suma que figura en la parte izquierda de la formula a demostrar, tendremos: Sn+l

1= (A-B)(b-e)-(B-C)(a-b) (a-b)(b-e)(a-e) , m = (A - B)(b 2 _e 2 )_ (B -C)(a 2 _b 2 ) (a-b)(b-e)(e-a) , n = a2 (Be -Cb)+ a(Cb 2-Be 2)+ A(Be 2-Cb 2) (a-b )(b -e )(e - a) . Observacion: Para x = a, x = b y x = e, el polinomio buseado rex) toma respectivamente los valoresA, B y C. Es faeil eomprobar que tal polinomio, no superior al segundo orden, es el siguiente: A (x-b)(x -c) B (x- a)(x -c) C (x-a)(x -b) (a - b)(a - e) + (b - a)(b - c) + (e - a)(e - b

r (5)

Puesto que el sistema (4) tiene una sola solucion, entonees, existe solamente un polinomio con la propiedad indicada y, por eonsiguiente, r (x) coincide con el polinomio (5).

= Sn + (n + I);n + 2) _ (11 + I)(n: 2)(11 + 3) =

= (n+l)[(n+l)+I][(n+I)+2] 6 De aqui, de acuerdo con el metoda de induccion matematica, se deriva la validez de la formula para cualquier valor entero positivo de n.

267. SeaSn lasumaquefiguraen la parte izquierdade la formula. Para n = 1, ambas partes de la formula coineiden. Demostremos que si la formula esjusta para cierto numero n, entonees sera tambien para n + I. Tenemos:

('1)2_Sn+l -S - n + 11 T

n(n+l)(2n+l) + (n + 1)2_6

= (n+l)(2n 2+7n+6)= (11-+-1)[2n(n+ 2)+3(n+ 2)] = 6 = (n + I)[(n + 1)+ 1][2{n + 1)+ I]

6

6 Por consiguiente, la formula es justa para eualquier n entero positivo.

ALGEsRAIResoluciont:s y Soluciones

268. Paran= 1es facil convencersede lajustezade laafinnaci6n. Supongamos que la f6nnula seajusta para cierto valor de n ~ 1. Designemos por S" la suma que figura en la parte izquierda de la

f6nnula. Tenemos: I

199

(coscp + isenCPt+! -(cosncp + i sen ncp)(coscp+ i sencp) =

= (cosncp coscp-senncp sen cp)+ i(cosncp sencp+ senncp coscp) = = cos{n + 1) cp + i sen{n + 1) cp. Por consiguiente, la f6nnula es justa para cualquier valor entero positivo de n.

Sn+!=Sn+(n+ln+2 )( )(n+3 )= _ n(n+3) , 1 - 4(n+l}(n+2)T (n+l)(n+2)(n+3)"

270. Es evidente que a + b = 1 y ab = -1. Aprovechando este hecho se puede escribir que

Deaqui

41_

n3 + 6n 2 + 9n + 4 = (n + 1)(n 2 + 5n + n 4(n+l)(11+2)(n+3) 4(n+1)(n+2)(n+3) _ (n+l)[(n+l)+3] - 4[(n+l)+I][(1I+l)+2]"

S +1 =

J5

-

- ---

~5

obien

Por consiguiente, la f6nnula es justa para cualquier valorentero positivo de n. 269. Por 10 visto, la f6rmula es justa para n = 1. Supongamos

que ella sea justa para cierto valor de n ~ I, es decir,

de don de

De aqui se deduce que si para cierto valor de n, los numeros an- I yan son enteros v positivos. entonces, tambien an+l y' por consiguiente an+2' all+3,· .. seran numeros enteros y positivos. Pero como a] = 1 y a 2= I, por 10 tanto, para 11 > 2 todos los valores de an seran enteros y positivos. . I " '

(cos cp + i sen

cp) n = cos ncp + i sen ncp.

-

(1)

Para convencerse de la justeza de la f6rmula para 11 = 1, multipliquemos am bas partes de (1) por cos cp + i sen cp. De acuerdo con la regIa de multiplicaci6n de numeros complejos, obtendremos:

200

PROBLEMAS DE MATEMAnCAS ELEMENTALES

271. Para n = 1 la desigualdad es justa. Supongamos que sea justa para cierto valor de n. Multiplicando ambas partes deesta desigualdad por 1 + an+l > 0, hallaremos que

(1 + a1)(1 + a2 )··· (I + an )(1 + an+1)~ ~ (1 +a1+a2 + ... +aJ(I +an+ 1 )=

Abriendo aqui los corchetes, transformemos cada uno de los

n + 1 sumandos por la f6rmula

(k )(a)k(b )n-k(a + b- n) = (k )(a)k(b )n-k[(a - k)+ (b - n+ k)] = =(k )(a)k(a - kXb )n-k + (k )(a)k (b )n-k(b - n+ k) = = (k}(a)k+l(b)n-k + (k)(a)k(b)n-k+l (k=O, 1, ... ,n). Como resultado obtendremos:

Puesto que la sumaa1an+l+a2an+l+'" +apn+I>O, entonces de aqui se deduce que la desigualdad tambien es justa para n + 1. 272. Ante todo nos convencemos de que la f6rmula es valida para n = 1. En efecto, para n = 1 la f6rmula tiene la forma

Valiendose ahora de la definici6n de patencia generalizada, se hace evidente que ambas partes de la f6rmula (1) son iguales a a+b y, par consiguiente, efectivamente tiene lugar la igualdad. Supongamos ahora que la f6rmula es valida para cierto valor de n y demostremos que sera tambien valida para n + 1. De la defmici6n de potencia generalizada tenemos:

(a+btl =(a+bt (a+b-n)= [(~)(a)o(bt +(~ )(a~(b )n-l + ... + +(~)(a)k(btk + ... + (~)(at(b )o](a +b - n)

(a+ b)n+l = (o)(aA (b)n + (o)(a)o (b)n+l + (r )(a)2 (b )n-l + + (r)(a)l (b)n +"'+(I:Xa)k+l (b)n-k + (k)(a)k (b)n-k+l + ... + +(~)(a)n+l (b)o +(~)(a)n (b)l' Despues de reducir los terminos semejantes tendremos:

(a +b)n+! = (o)(a)o (b )n+l + l(o)+ (r )J (aA (b)n + + [(1 )+ (~)](a)2 (b )n-l + ... +[(1:)+ (k:J (a )k+l (b )n-k + ... + + [C~l)+ (~)](a)n (b A+ (~)(a )n+l (b )0' Aprovechando, a continuaci6n, el hecho de que

=

y la identidad facil de comprobar

AWEBRAIResoluciones y Soluciones

obtenemos:

De acuerdo con la f6rmula( 4) en lapagina 42 eI valor minimo de ret) y, porconsiguiente, de ret), se obtiene en el momento de tiempo

(a+bLl =(n;I)(aMb)n+l +(ntl)(a~(bt + +{n;l )(a)2(b )n-l +... +(~:: )(a )k+Jb )n-k +... + +(n:l )(a t(b h+ + (~:: )(a t+l (b )0' Porconsiguiente, hemos demostrado que si la f6rmula, expuesta en la condici6n del problema, es valida para cierto valor de n, entonces es tambien valida para n + 1. Pero ella es justa para n = 1, por consiguiente, de acuerdo con el metodo de inducci6n matematicacompleta, es valida para todos los val ores enteros positivos de n. 273. Sea r(/) la d istancia entre los trenes en el momento de tiempo I. Entonces

r2(t)=(a-v1t)2 +(b-v2t)2 = (V12 +vnt 2- 2(av1 +bv2)t + a2+ b2. Observemos que si r(t) tiene el valor minimo para 1 = to' entonces, tambien r(t) adquiere el valor minimo cuando t = to' Esjusto y viceversa. El problema se reduce a la determinacion del valor minimo del trinomio cuadrado r (t).

201

to =

avl + bV2 2 2' VI + V2

Valiendonos acontinuacion de la formula (3), hallamos la distancia minima entre los trenes:

r{to) =

~vl +vi

.

274. En el momenta de tiempo I el coche se encuentra ala distancia de 40 t Ian del punto A y la motocicleta a la distancia de 3; (2 + 9 Ian del mismo punto. Por consiguiente, la distancia entre ellos es igual al valor absoluto de la diferencia 16P + 9 «40t. Designando esta diferencia por y(t), tracemos el gratico del trinomio cuadrado y(t) (fig. 4). Este grafico representa una parabola que, para los valores

II

= -i-

Y t2

= 2-i-,

intersectael

eje t. Del grafico esta claro que la ordenada de mayor valor absoluto y, para la condicion 0::;; t ::;; 2, corresponde al vertice de la parabola. El vertice de la parabola se encuentra en el eje de simetria que cruza el eje ten el punto II + 12 5 to = - - = - . 2 4

202

PROB LEMAS DE MATEMATICAS ELEMENTALES

As! pues, la distancia maxima entre el tren y la motocicleta se alcanza al cabo de 1 hora 15 minutos despues de iniciar el movimiento y es igua\ a 16 kIn. 275. Designemos la expresi6n que se analiza por y y transforme-mosla de la sigu iente manera:

y

y=21 og 4 x+ ~

+ 1221og2 x(1Iog8- 2 1ogx )=

276. Primera resolucion. Por 10 visto es suficiente examinar solamente los valores positivos dex. De acuerdo con laconocida desigualdad (3), pag. 20 tenemos:

ax 2 +b - - 2

~

1~2'l ax b

~

= x -v ab.

(1)

Por consiguiente, para todos los valores de x > 0 y

=

x ax 2

+b

~

x I

2x\l ab

= ------,====. 2-J ab

(2)

t

=llogZ x(zlogZ x-12 2 1ogx+36)= =zlog2 x(6- Z1ogx)2 .

Hagamos a continuaci6n 2 10gx =z, de modo que O~z~6. Entonces, el problema se reduce a la determinaci6n del valor maximo de lavariable

Es suficiente hallar el valor maximo de z(6-z) con la condici6n de que 0 ~ z ~ 6, puesto que cuanto mayor es un numero positivo tanto mayor es el cuadrado de este numero. El trinomio cuadrado z(6-z)=- {z- 3)2+9 a1canza su valor maximo paraz=3.As! pues, el valor maximo se consigue cuando z = 3 yes igual a 81.

Puesto que (1) pasa a ser una igualdad cuando ax1

= b,

I.

entonces, para Xo =~! tenemos: 1

Yo

= 2-J ab .

(3)

En virtud de (2) este es precisamente el valor maximo de la funci6n. Segunda resoluci6n. Resolviendo la relaci6n

y=

x 2

ax +b

(4)

respecto a x, obtendremos:

1 ± ~I- 4aby2 x=-- - - - . 2ay

(5)

ALGEBRAIResoluciones y Soluciones

De la formula (5) se desprende, que para todos los val ores

x2 + 1 r;:; --:2:-2+2,, 2 x+1

reales de x debe cumplirse la desigualdad 1 - 4aby2 :2: O. De

203

(2)

Yel signo de igualdad en esta formula tiene lugar cuando

aqU)

(6)

x=J2-l. Puesto que para cierto valor real de Xo la funcion (4) adquiere elvalor Yo

= 2"J~a b (de la formula(5)hallamos que

Xo

= ~ (f), a I

entonces, en virtud de (6) este valor es el maximo.

278. Tomemos el eje numerico y marquemos en ellos puntosA, B, C y D correspondientes a los numeros a, b, C Y d. EI punto con la abscisa variable x 10 designaremos por M (fig. 5) Examinemos los cinco casos siguientes.

277. Por medio de las transformaciones evidentes, obtenemos:

~2 + 1 = x-1 :. _2_ = -2 + [x + 1 + _ 2_ ] . x+l

x+l

A

B

C .

D

x

a

b

c

d



x+l

.

M



..

Fig. 5

En virtud de la desigualdad (3), pag. 20 1) x::;; a; entonces

x+ 1+ _ 2_ :2: 2 /(x+ x+l ~

1)-{

2 ) = 2J2, x+l

(1)

con la particularidad de que el signa de igualdad en (1) tiene lugar solamente en el caso cuando 1+ x

2 -

= - -1-' x+

es decir, para Xo

= -02 -

As) pues, para todos los valores de Xo =:2: 0

1.

AE + BF > AC + BD.

Fig. 106

AM +BM

Fig. 107

>AB >1 AM -BM i,

puesto que las hipotenusasAEy BF en los triangulos recmngulos AECy BFD son mayores que los catetosACy BD. En el caso b) el segmentoAB se encuentra dentro del angulo CAC' (fig. 109). Tracemos por el punto B una circunferencia concentrica a la dada. Supongamos que esta circunferencia corta a AC y a AC' en los puntos EyE'. Entonces, EC=BD y AE>AB. Por consiguiente,

254

PROBLEMAS DE MATEMAnCAS ELEMENTALES

de donde,

AB < AE == AC - EC == AC - BD.

x

395. Introduzcamos las siguientes anotaciones (fig. 110): LPCM== LQCN= a, LNML=LNKL==y, LLCP=LQCK= jJ, QC=x,

PC=y,

2

=

a 2 senysen(a+f3+y) sen a sen f3 + sen ysen (a +f3+ y)

.

Amilogamente se determina que

AC=CB=a.

y

2

==

a 2sen y sen (a + f3 + y) sen a sen f3 + sen y sen (a + f3 + y) .

As! pues, x = y.

396. Sean B 1, B 2 , B3 Y B410s puntos medios de los arcosA1A z' (Fig. Ill). Sea, ademas, a 1 el angulo central correspondiente al arcoA,B;Ci=l, 2, 3, 4). Designemos por

ANy MN> AM, puesto que MN es la hipotenusa del triangulo AMN. Por eso 2MN> AN + AMy, en virtud de (1) tenemos que 3MN> AN + NC + AM+ MB = AB + +AC. Por consiguiente,

MN> AB+AC. 2

Fig. 116 Fig. 117

GEoMETRiAIPlanimetria (Resoluciones y Soluciones)

402. Val iendonos de las denotaciones introducidas al resolver el problema 324, hallamos: n2

bd) = ab + cd(ac+, bc+ad

m2

bd) = bc + ad (ac+. ab+cd

Dividiendo miembro a miembro estas igualdades, obtenemos:

n

m

ab+cd bc+ad

403. Sea ABC un triangulo regular con los lados ay rJ' r2 y r3las distancias desde el punto M de la circunferencia circunscrita al triangulo hasta los vertices de este (fig. 117). Observemos, al principio, que para la posicion del punto M dada en la fig. 117 tendremos que

257

404. Supongamos que ellado AB del cuadrilatero ABCD cruza ala circunferencia y que los lados BC, CD y DA hacen contacto con ella en los puntos E, Fy G(fig. 118). Puesto que CE=CFy DF=DG, entonces, la desigualdad AB+CD>BC+DA es equivalente a la desigualdad AF>BE+AG, que fue demostrada en la resolucion del problema 394. 405. Supongamos que ellado AD del cuadrilatero ABeD no corta a la circunferencia y que los lados BC, CD y BA hacen contacto con esta en los puntos F, E Y G (Fig. 119). La desigualdad

AD+CB < DC + BA es equivalente a la desigualdad

AD 2, ambas raices de la ecuaci6n cuadnitica tienen senti do, 493. El radio r de cada una de las ocho esferas inscritas 10 hallaremos examinando el triangulo AOe en el plano que pasa por los centros de estas esferas y el centro 0 de la esfera S (fig. 205, a)). Tenemos:

1 +sen a r = H tg a = R . - - - cos a Por consiguiente,

AB

r

1t

AO

R-r

8

-=--=sen-.

k=!.

(1+sena)3 =!. (1+sena)2 4 sen a (I - sen 2 a) 4 sen a (1 - sen a

Sustituyendo aqui sen a = 1 respecto a ~ = Z :

i

r

De aqui

0,

r = R _ -----'8 "'----sen 2: +! 8

obtenemos una ecuaci6n

k-! (2-zY - 4

1t

sen-

Trazando una secci6n que pase por el centro 0 de la esfera

(I-z)z

S, el centro O[ de la esfera S[ y los centros de las dos esferas

despues de las simplificaciones correspondientes,

opuestas de radio r (fig. 205, b),

z2(4k + 1)- 4(k + l)z + 4 = O. Resolviendo esta ecuaci6n, obtenemos: Z 1,2

2(k + 1)±2~k(k -2) = --"-----'-----'-------'-------'-4k+! '

Definitivamente hallamos:

V;

(1) b)

a)

zi, (3 2

zl,

J

V2 = 4- Z 7,2 ~-z~~l

Fig. 205

del triangulo rectangulo A001' obtendremos:

GEOMETRiAiEstereometria (Resoluciones y Soluciones)

313

obtenemos la ecuaci6n

R-x = x0}3,

o bien de donde De aqui

R-r p=R·--, R+r o bien

R

p=R·---1[ 2 sen- + 1 8

495. Supongamos que sea r el radio de la base de cada uno de los dos con os inscritos. Su parte com un se compone de dos conos truncados iguales. Designemos por r l Yr2 los radios de las bases superior e inferior respectivamente del conn truncado y por H su altura. La relaci6n buscada de los volumenes es

494. Puesto que las esferas inscritas son iguales entre sl, sus centros equidistan del centro 0 de la esfera S. Por consiguiente, el centro de simetria del cuba indicado en las condiciones del problema coincide con el centro 0 de la esfera S (fig. 206). Sea x el radio buscado de las esferas. Es facil ver, que entonces la arista del cuba sera AB=2x, y la mitad de la diagonal del cuba

A

AO =CO-CA =R-x. Fig. 206

Puesto que, por otro lado,

AO =!. 2x0}3 2

'

Fig. 207

De la semejanza de los triangulos AQZ, AOS y APC (fig. 207) tenemos: !l= R-H r2 R R y r h

314

PROBLEMAS DE MATEMAT]CAS ELEMENTALES

Puesto

que,

ademas,

H=h-R y r=-JR 2 -H 2 =

De estas dos ecuaciones haHamos.

= -J2Rh - h 2 , las dos igualdades anteriores permiten expresar r]

y r 2 en funci6n de R y h:

fj

y, por consiguiente,

2R-h = r2 R

Ya que de la condici6n del problema

i

= k, entonces De las ecuaciones (1) y (2) obtenemos:

Por esta raz6n, el area buscada es

496. Supongamos que los radios de las secciones circulares con las areas S] y S2 sean iguales a RI y R 2 , Y que las distancias desde el centro de la esfera hasta dichas secciones sean respectivamente iguales a II y 12 (/\ < 12)' Designemos por R el radio de la esfera, por r el radio de la secci6n buscada y por I la distancia desde esta secci6n hasta el centro de la esfera. Entonces, (fig. 208), (1) y

497. Designemos por r el radio buscado de la base del cono. Examinemos la figura obtenida en la secci6n trazada por el centro de una de las esferas y el eje del cono (fig. 209). Observemos que la distancia entre los centros de dos circunferencias en contacto es igual a 2R. Valiendonos del hecho, facil de demostrar, de que el centro de la base del cono A equidista de los tres puntos de tangencia de las esferas con el plano P, hallamos:

GEOMETRiAiEstereometria (Resoluciones y Soluciones)

De aqui , siendo q = 2, obtenemos que x =

AD= 2J3 R. 3

consiguiente, r x

=

_2J3 (q-I)=t ,J9q 2- 18q+12

Dado que 0 < x
AK = 2MN (aqu! tiene importancia que D no se encuentra sobre la recta AK, de 10 contrario, tendrfamos que haber puesto el signo ~). As! pues, BC+AD>2MN, 10 que era necesario demostrar.

521. Sean A, B, C Y D puntos arbitrarios que se encuentran en las aristas del angulo tetraedrico con el vert ice E (fig. 234). Demostremos, por ejemplo, que

LCED < LCEA + L AEB + L BED,

(1)

Tracemos el plano CEB. Segun la propiedad de los angulos pianos de un angulo triedrico

L CED < L CEB + L BED.

(2)

y por la misma causa

LCEB < LCEA + LAEB.

(2)

De las desigualdades (2) y (3) se desprende (1). De este modo, la desigualdad (I) queda demostrada.

522. Supongamos que sea dado el angulo tetraedrico convexo con el vertice S (fig. 235). Prolonguemos los pianos BSC y ASD hasta que se crucen por la recta II' Luego, prolonguemos los pianos ASB y DSC hasta que se crucen por la recta Las rectas y evidentemente, no coinciden (de 10 contrario, todas las caras en su prolongaci6n pasarian por una misma recta). Designemos por P el plano en el que se encuentran las rectas y Valiendonos de la convexidad del angulo tetraedrico, es f-li-1.

I

TRlGONoMETRiAiResoluciones y Soluciones

574. Haciendo sen x - cos x = I Y valiendonos de la identidad (sen x - cos x)2 = 1- 2 sen x cos x, escribamos la ecuacion inicial en la forma 12

+ 121 -13 = O.

Esta ecuacion tiene las rakes I,

l=senx-cosx=J2sen(x-~),

= -13 Y

12

= 1. Pero,

Esta ecuacion es del mismo tipo que la del problema 574. Sustituyendo sen x + cos x = I, la ecuacion (l) se lleva a la ecuacion cuadrada 12 + 41 + 3 = 0, cuyas ralces son I, = -1 Y 12 = -3. Puesto que I sen x + cos xl:s;; J2, a la ecuacion inicialla pueden satisfacer solamente las rakes de la ecuacion sen x + cos x

dedonde I 11:s;; J2, por

consiguiente, la raiz I, = -13 puede no examinarse. Por esta razon, la ecuacion inicial se reduce a la siguiente:

349

= -1

(2)

Resolviendo la ecuacion (2), obtenemos: n: 2 x 2 = (2k + 1)n:.

Xl

= -- + 21m,

La segunda serie de rakes se debe despreciar, puesto que sen x2 = 0 y la ecuacion inicial pierde el sentido. Respuesta: x,

n:

= n: + 21m, x2 = 2" + 21m.

Respuesta x

575. Transformemos la ecuacion dada a la forma

576. La ecuacion dada pierde el senti do cuando x cuando x"* 1m puede ser escrita en la forma

2 cos 2 ~(2 + sen x)+ sen x = O. 2 Utilizando la formula 2 cos 2 ~ . . 0 btenemos: 2 parentesls,

= I + cos x y abriendo los

2 + 2 {sen x + cos x)+ sen



cos x

= O.

n:

= - -2 + 2kn:. .

(1)

= 1m,

y

Pasando todos los terminos de la ecuacion a la parte izquierda y descomponiendolos en factores, obtenemos: (cosx-sen x) (sen 2 x+cos2 x+sen x cosx+sen x+cosx)= o.

350

PROBLEMAS DE MATEMATlCAS ELEMENTALES

De aqui se desprenden dos posibilidades: a) sen x - cos x = 0, entonces, x

1

2 (sen 3x cos

= ~4rr + 1m.'

b) sen 2 x + cos 2 X + sen x cos x + sen x + cos x

2x - cos 3x sen 2x) cos 2x = = sen 2x (sen 2x sen 3x + cos 2x cos 3x).

(I)

= 0.

(2)

de donde x

= krr.

578. La ecuaci6n dada se puede escribir en la forma La ecuaci6n (2) es analoga ala examinada en el problema 574 y tiene las soluciones

rr 2

x2

= - ~ + 21m

(3)

X3

= (2k + I)rr

(4)

y

3(COS 2x _ cos 3X) = sen 2x + cos 2x sen 2x

sen 3x

cos 2x

sen 2x

o bien

Pero, los val ores de x que figuran en la f6rmula (4), no son ralces de la ecuaci6n inicial, puesto que, siendo x = nrr la ecuaci6n in icial pierde el sentido. Por consiguiente, la ecuaci6n tiene las rakes determinadas por las f6rmulas (I) y (3). 577. Escribamos la ecuaci6n de la manera siguiente:

3 sen x sen 2x sen 3x

sen 2x cos 2x

Observemos que esta ecuaci6n tiene sentido si sen 2x;;t:. 0,

sen 3x;;t:. 0,

cos 2x;;t:. 0.

Para los val ores de x, para los cuales la ecuaci6n (1) tiene sentido, 3 sen x cos 2x = sen 3x, o bien sen x

Reduciendo los quebrados a un comun denominador y liberandonos de este ultimo, obtendremos la ecuaci6n

(3 - 4 sen

2

x-

3cos 2x) = 0,

o bien 2 sen 3 x

= 0.

TRlGONoMETRiAiResoluciones y Soluciones

Puesto que la ultima ecuaci6n es equivalente a la ecuaci6n sen x = 0, entonces, en virtud de la observaci6n hecha mas arriba, la ecuaci6n inicial no tiene rakes. 579. Escribimos la ecuacion en la forma 6(tg x + cotg 3x) = tg 2x + cotg 3x, despues de 10 cualla transformamos de la siguiente manera: 6 (sen~ + cos 3X) = sen 2x + cos 3x cos x sen 3x cos 2x sen 3x

Durante la resoluci6n se realiz61a multiplicaci6n de ambos miembros de la ecuaci6n por cos x cos 2x sen 3x. Pero, es facil ver, que para ninguno de los valores hallados de x este producto se reduce a cero. Por consiguiente, todos los valores hall ados de x son rakes de la ecuaci6n inicial.

580. Reduciendo los quebrados que figuran en el segundo miembro de la ecuaci6n a un comun denominador y empleando la f6rmula

as _b 5 =(a-b)(a 4 +a 3b+a 2 b 2 +ab 3 +b 4 ), obtenemos:

o bien

6cos2x cos x sen 3x

sen x cos x{sen x - cos x) (sen 4 x + sen 3 x cos x +

cosx cos 2x sen 3x '

4 x ) =sen x-cosx. +sen-" x cos- x+sen x coso'x+cos

6 cos 2 2x = cos 2 x; 12

cos 2

351

De aqul se desprende, que a bien

2x - cos 2x -1 = 0.

(I)

sen x - cos x = 0.

Resolviendo la ultima ecuaci6n, hallamos: o bien

1± 7

{4

24

1 1 x = ±--arccos- + 1m' 2 3 '

-'

1

2) cos 2x = - --, 4

x = ±larccos(

'

cos 4 X + sen 2 x cos 2 x )-1 = 0,

de donde I 1) cos 2x =-.., ,

3

sen x cos x\sen x + sen x cos x + sen x coso x +

cos2x=--,

-l)

(2)

Transformemos, ahora, la ecuaci6n (2), aprovechando que sen 4x + cos 4x = (2 sen x + cos 2x )2 - 2 sen 2x cos-' x, + 1m.

352

PROBLEMAS DE MATEMATICAS ELEMENTALES

yque,

2 X

sen 3 X

COS X

+ COS 3 X sen

X

= sen X COS X.

Haciendo, ademas, en la ecuaci6n (2) sen escribamos la ecuaci6n (2) en la forma 3

x x tg- - cotg 2 2

X

cos

X

= y, (3)

o (despues de descomponer el primer miembro en factores) en la forma

Si Y = I, es decir, si sen X cos X = I, entonces, sen 2x = 2, 10 que es imposible. Si y = -I, entonces, sen 2x = -2, 10 que es tambien imposible. Asi pues, la ecuaci6n (2) no tiene rakes. Por consiguiente, las raices de la ecuaci6n inicial son las rakes de la ecuaci6n

*

= + 1tn.

Por consiguiente, si x"* krt y x"* + m1t (k y m son numeros enteros arbitrarios), el segundo miembro de la ecuaci6n es igual a - 2 sen x cos x. El primer miembro de la ecuaci6n no tiene sentido cuando x = + krt y x = + I· (i = 0, ± 1, ± 2, ... ), Y para los demas val ores de x, es igual a la - tg x, puesto que

*

i

tg (x -

i

~ Jtg ( x + ~ J= tg ( x - ~ Jcotg [ %- (x + ~ J]=

= tg ( x -

~ Jcotg ( x - ~ J= -1.

i

Asi pues, si x"* + nm y x"* ecuaci6n inicial tiene la forma

581. EI primer miembro de la ecuaci6n pierde el sentido cuando x = k1t Y cuando x = + m1t, puesto que siendo x = 2lJr la funci6n cotg~ es indeterminada, siendo x = (21 + 1)1t es indeterminada la funci6n tg y siendo x = + m1t, el denominador del segundo miembro se hace igual a cero. Si x"* krt tenemos:

i

i

2 cos x sen x

i

2

Y -y -y+l=O

(1), es decir, x

2X

sen --cos 2 2_ = x x sen-cos2 2

1,

tg x

* i, +

entonces la

= 2 sen x cos x.

Esta ecuaci6n tiene las rakes

1t 1t x=krt y x=-+I-. 4 2 De aqui se desprende que la ecuaci6n inicial no tiene rakes.

TRlGONoMETRiAiResoluciones y Soluciones

582. Multiplicando el segundo miembro de la ecuaci6n por sen 2 x + cos 2 X = 1, la llevamos a la forma (I-a)sen 2 x -sen x cos x-(a + 2)cos 2 x = 0.

(1)

"*

Supongamos, al principio, que a l. Entonces, de (1) se desprende que cos x 0, puesto que, en el caso contrario, tendrfamos que sen x = cos x = 0, 10 cual es imposible. Dividiendo ambos miembros de (1) por cos 2 x y haciendo tg x = t, obtendremos la ecuaci6n

"*

(I-a)t 2 -t-(a+2)=0.

(2)

La ecuaci6n (1) tiene soluci6n cuando, y s6lo cuando, las rakes de la ecuaci6n (2) son reales, es decir, cuando su discriminante es

D

= 4a 2 - 4a + 9 ~ 0.

(3)

Resolviendo la desigualdad (3), hallamos:

-

-JiO + I -::;a-::; -JiO - 1. 2

2

(4)

Sean t, Y t2 las rakes de la ecuaci6n (2). Entonces, las correspondientes soluciones de la ecuaci6n (1) tienen la forma Xl

= arctg tl + 1m,

X2

= arctg t2 + krr.

Examinemos ahora el caso en que a

= 1.

353

En este caso, la ecuaci6n (1) se escribe en la forma cos x (sen x + 3 cos x) =

°

y tiene las soluciones siguientes: rr

xl

= - + 1m, 2

x2

= -arctg 3 + 1m.

583. Empleando las f6rmulas 4

sen x

_ (

2 1 + cos 2x cos x = - - - 2

1 - cos 2X) 2

-I

'

" 2 y haciendo cos 2x = t, escribamos la ecuaci6n dada en la forma siguiente:

t2

-

6t + 4a 2

-

(1)

3 = 0.

La ecuaci6n inicial tendni soluci6n solamente para tales valores de a, para los cuales las rakes t, y t2 de la ecuaci6n (1) sean reales y, por 10 men os, una de estas rakes, por su valor absoluto, no sea mayor que la unidad. Resolviendo la ecuaci6n (I), hallamos: r:::-J

tl =3-2'J3-a-,

r-2

t2 =3+2')3-a .

Por consiguiente, las rakes de la ecuaci6n (1) son reales si (2)

354

PROBLEMAS DE MATEMAnCAS ELEM ENIALES

Si se cumple la condicion (2), entonces 12 > I, y por esta razon, esta raiz puede ser despreciada. De este modo, el problema se reduce a la determinacion de aquellos valores de a que satisfacen la condicion (2), para los cuales III I:::; 1, es decir, para los cuales - 1 :::; 3 - 2 -J 3 - a 2

:::;

1.

(3)

584. Transformemos la ecuacion dada, multiplicando ambos miembros por 32 sen ~. Empleando unas cuantas veces la formula sen a cos a = sen 2a, obtendremos:

1

32

o bien

1tX

sen -

De (3) hallamos que

-4:::;-2 -J3-a 2 :::;-2,

1tX

sen - 1tX = sen31 31

2

33

COS -

62

x2 = -31 (2n + I) 33

(4)

Puesto que la desigualdad 2? ~3 - a 2 se cumple siendo

Ia 1:::;J"3, entonces, el sistema de desigualdades (4) se reduce a la desigualdad

sen -

31

?I,

de donde hallamos que I a I :::;-fi. Asi pues, la ecuacion inicial es soluble si I a I :::; -fi, y tiene la solucion

(n = 0, ± 1, ± 2, ... ) .

Puesto que durante la resolucion se realizo la multi plicacion de ambos miembros de la ecuacion dada por el factor 32 sen ~~, que puede reducirse acero, la ecuacion (1) puede tener raices ajenas para la ecuacion inicial. Seran raices ajenas unicamente las raices de la ecuacion 1tX

-J3-a 2

(1)

De aqui hallamos dos series de raices:

de donde

2 ?-J3 - a 2 ? 1.

1tX = 0.

=0.

(2)

que no satisfagan a la ecuacion inicial. Las raices de la ecuacion (2) se dan por la formula x

= 31k

(k = 0, ± I, ± 2, . .. )

(3)

y, como es f£lcil ver, no satisfacen a la ecuacion inicial. Por esta razon, de la serie hallada de raices de la ecuacion (1),

TRiGONoMETRiAIResoluciones y Soluciones

deben ser excluidas todas aquellas que tienen la forma (3). Para las rakes de la primera serie esto conduce a la ecuaci6n 2n = 31k, que es posible solamente para los valores pares dek, es decir, para k = 21 y n = 311 (I = 0, ± I, ± 2, ... ). Para las rakes de la segunda serie amilogamente obtenemos la igualdad ~j (2n + 1) = 31k 0 bien 2n + 1 = 33k, que es posible unicamente cuando k es impar, 0 sea, cuando k=2/+1 y n=33/+16(/=0,±I,±2,... ). Para las rakes de la segunda serie amilogamente obtenemos la igualdad *(2n + 1) = 31k 0 bien 2n + 1 = 33k, que es posible unicamente cuando k es impar, 0 sea, cuando k = 21 + 1 y n=33/+16(/;t:O, ±I, ±2, ... ).

Xl

I

= 0, ± 1, ± 2, ...

1 -J3 -J3 1 -cos 7x+-sen 7x= - cos5x+-sen 5x, 2 2 2 2 o bien

es decir,

1t 3

-13 = 2kn,

0

1t 6

1t 3

-+ 7x- - -5x = 2kn o bien

2: + 7 x + 2: + 5x = (2m + 1)1t. 6

X

= ~(12k + 12

x =..2:.(4m+ 24

585. Escribamos la ecuaci6n de la manera siguiente:

1t 1t sen - cos 7x + cos-sen 7x 6 6

= sen 13 cuando, y s610 cuando, 0 bien bien a + 13 = (2m + 1)1t (k, m = 0, ± 1, ± 2, ...) . Por consiguiente, Pero, sen a

a

3

As! pues, las rakes de la ecuaci6n serlin:

As! pues, las rakes de la ecuaci6n inicial son: = 2n, donde n ;t: 31/, } 31 x2 = -(2n + 1), donde n;t: 331 + 16. 33

355

1t sen 5x, 3

= sen - -cos 5x + cos -

1),1 I).J

(k, m = 0, ± I, ± 2, ... ).

586. Puesto que el primer miembro de la ecuaci6n es igual a 2 - (7 + sen 2x) (sen 2 X

-

sen 4 x) =

= 2 - (7 + sen 2x) sen 2 X cos 2 X = 2 - (7 + sen 2x )3. sen 2 2x, 4 haciendo t

= sen 2x, escribamos la ecuaci6n en la forma (I)

356

PROBLEMAS DE MATE~I."TICAS ELEMENTALES

La ecuaci6n (1) tiene la raiz evidente rakes se hallan de la ecuaci6n

II

= 1.

t 2 + 8t + 8 = O.

Sus otras dos

Puesto que siempre Isen (x +

- Yl; en el tercer caso, la soluci6n 1t + xl> 1t + Yl; en el cuarto, la soluci6n 1t - Xl' 1t - YI. Por consiguiente, el sistema dado tiene por

y) = 2 (a 2 + b2 )- I,}

(a 2 + b2 )cos{x + y)= b 2 _ a 2 , tgx·tgy=c, equivalente al sistema (1). Si a 2 + b 2 = 0, la segunda ecuaClOn se satisface cualesquiera que sean los valores de x e y. De la primera ecuaci6n obtenemos:

TRIGONoMETRiAlResoluciones y Soluciones

x- Y

= 1t + 21m

(k = 0, ± 1, ± 2, ... ).

377

evidentemente, es justa, y la desigualdad (3) es equivalente a lasiguiente:

la tercera ecuaci6n nos da: tg(y + 1t + 21m}tg y

=c

o bien

Representemos el primer miembro de la tercera ecuaci6n del sistema (I) en la forma siguiente:

tg 2 Y = c. Esta ultima ecuaci6n tiene soluci6n para cualquier valor de c ~ 0. Si a 2 + b 2 7= 0, tenemos:

~Icos{x- y)-cos{x+ y)1

tg x tg Y

2 = sen x sen Y = ~--------

cosxcosy

1 -Icos{x- y)+cos(x+ y)1 2

(5)

y sustituyamos en (5) cos (x + y) y cos (x - y) por sus valores de (2). Como resultado obtendremos que la soluci6n del sistema (2) satisfani ala tercera ecuaci6n del sistema inicial, S)

Este sistema tiene soluci6n cuando, y s610 cuando,

(2

_ 2 ~a + b

2)

b2 - a2 2 - 1- a 2 + b2 _ (a2 + b2) _ b 2

c - b 2 - a 2 + 2 (a2 + b 2)_ 1 - (a 2 + b 2 a 2 +b 2

Y_a 2 .

Hemos lIegado al siguiente resultado: el sistema dado tiene por 10 menos una soluci6n en dos casos: La desigualdad (4), con la condici6n de que

2

( 2 a + b2 ) b2 1) 0< a 2 + b 2 ::::; 1 y C = 2 ; (a 2 + b 2 ) _ a 2

2) a

= b = 0 y c es cualquier numero no negativo.

378

PROBLEMAS DE MATEMATICAS ELEMENTALES

3. Funciones trigonometricas inversas

Por consiguiente,

619. De la definicion de los valores principales de las funciones trigonometricas inversas se desprende que

arcsen ( cos

3:

1t )

= arcsen [ sen ( -

1~) 1= - 1~ .

arccos{cos x) = x, si 0::::; x::::; 1t. Con el fin de utilizar esta formula, sustituyamos sen (- ~) con ayuda de las formulas de reduccion al coseno del angulo inc1uido entre 0 y 1t. Escribamos las iguaJdades siguientes:

621.

Supongamos que sea

arctgt=u" arctgt=u 2 , TC

arctg* = u 3 , arctgt = u 4 . Es evidente, que 0 < U i < 4' i = = I, 2, 3, 4. Por eso,

En resumen obtenemos:

arccos[ sen ( -

~ ) ] = arccos (cos ~: ) = ~: .

Para la demostracion de la identidad es suficiente establecer que Puesto que

620. Por analogia con la resolucion del problema anterior tenemos:

622. Haciendo arcsen x = a, arccos x x

= sen a

y

x

= 13,

tendremos:

= cos 13 = sen ( ~ -13) .

TRlGONoMETRiAiResoiuciones y Soiuciones

Segun la definicion de los valores principales tenemos que

379

a) Si O:s: x :s: I, entonces O:s: a :s: ~ y O:s: f3:S: ~ (puesto que O:s: -11- x 2 :s: 1 ). Queda solamente convencerse de que

De la ultima desigualdad se deriva la desigualdad 1t

1t

1t

2

2

2

sen a

= sen f3.

tenemos que sen a

--:S: - -f3:S:-.

=i -

Por 10 tanto, a f3, puesto que los angulos a y ~ - f3 estan incluidos entre - ~ y ~ y los senos de estos angulos son iguales entre sl. La formula queda demostrada. 623. Aprovechando que arcsen x + arccos x = ~ (vease la resolucion del problema 622), transformemos la ecuacion a la forma (1)

-

tenemos que sen arcsen y 1-

= sen arcsen \/1 -

y(lyi:S:1)

-2

x

=

y,

~

= \/1 -

en particular, sen ~ =

Por consiguiente, para

x- .

O:s: x :s: 1, tiene lugar la formula arccos x -1 :s: x :s: 0,

"i S 1t -!3 ~ 7t. sen (1t -

481t 4 (1- 8a) < O.

= +~1 - x 2 .

Por otra parte, para todos los valores de

a) Si donde t = arcsen x. Siendo a < 312' el discriminante de esta ecuacion sera

D = 361t 4

Pero, en virtud de la desigualdad O:s: a :s: ~,

=

arcsen ~ 1 - x 2 .

entonces

Puesto que, ademas, sen a

f3) = sen f3 = ~1- x 2 ,

entonces a

=:

~1- x 2 y

= 1t - f3,

es decir,

para - 1 :s: x :s: 0 tiene lugar la formula Por consiguiente, las raices de la ecuacion (I) son irreales y por eso, para a < 312' la ecuacion inicial no tiene soluciones. 624. Hagamos arccos x

= a,

arcsen

,J 1 -

x2

= f3.

arccos x

= 1t -

arcsen ,j1 - x 2 .

625. Demostremos que arcsen (- x) = -arcsen x. Hagamos arcsen (- x) =: a; entonces - x = sen a y, segun la definicion de los valores principales,

380

PROBLEMAS DE MATEMAnCAS ELEMENTALES

1t

en virtud de (I), obtendremos:

1t

--:5:a:5:-. 2 2

(1)

I-x 2 sen a =--2 ' l+x

Puesto que sen (- a) = -sen a = x y, puesto que de la desigualdad (I) se deriva la desigualdad - ~ :5: -a :5: }-, entonces - a = arcsen x de donde a = -arcsen x,

0

de donde 1 x2 Y = arcsen (sen a) = arcsen ~ = I +x

sea,

arcsen (- x) = -arcsen x.

13.

(2)

Puesto que 0 < x < 1, entonces

Analogamente se demuestra la f6rmula arccos(- x) = = 7r - arccos x.

1t l+x 1t 1t - < arctg - - < - y - < a < 1t.

4

626. De la definici6n de los valores principales de las funciones trigonometricas inversas se desprende que arcsen (sen a) = a,

Si

SI

I-x

2

2

De aqu, se desprende que

-f +21m:5:

7r

- - O. -2 - sen x - --2[sen x + -1+-15)[ 1+-15 >1 2

y por eso,

= +Ft- x 2 , donde - 1 :::; x :::; 1.

1

+-J5

sen x+ - - >0. 2

Designemos y = +~1- x 2 ; entonces 0:::; y :::; 1. As! pues, hay que hallar la relacion entre arcsen y y arccos y siendo 0:::; y :::; 1. Estos dos angulos complementan uno al otro hasta ~ (vease la resolucion del problema 622). As! pues, arcsen cos arcsen x + arccos sen arccos x

(2)

Pero,

Aqu!, claro esta, es esencial que - ~ :::; arcsen x:::;~. Analogamente sen arccos x

381

1t

Por consiguiente, la desigualdad inicial es equivalente a la siguiente:

-J5 -1 -

senx> -

2

y tiene la solucion

= -. 2

21m +

cos 7x

0

1

bien sen 5x sen 2). > o.

Pero, cuando 0 < x < tenemos que sen 2x > 0 y, por consiguiente, la desigualdad inicial es equivalente a la siguiente: sen 5x > o.

Respuesta: 0 < x < 3- y %n < x
-1 0 b·len sen x > -1 . 4 2 n 5 Respuesta: "6 + 1m < x < "6 n + 1m. 633. Escribamos la desigualdad en la forma (cos x - sen x) [1- (cos x + sen

x) ]=

= 2 sen ~(sen ~ - cos~) (cos x - sen x) > o. 2

2

(1)

2

1>

Pero sen 0, puesto que 0 < x < 2n. Examinemos dos casos posibles, en los cuales se cumple la desigualdad (1).

Caso 1. cos x - sen x >

0,1

x x sen "2 - cOS"2 >

O.J

~

(2)

TRlGONOMETRiAiResoiuciones y Soiuciones

Segun la condicion del problema, 0 < x < 27t. Teniendo en cuenta este hecho, de (2) hallamos que la primera desigualdad se cumple cuando 0 < x < ~ 0 bien %7t < x < 27t, y la segunda, cuando ~ < x < 27t. Por . . conslgmente, en este caso, 4"5 7t < X < 2 7t.

1-1 > o. I -1-1

EI trinomio t 2 - I -1 tiene las rakes 1-.J5 2 y 1+.J5 2· Resolviendo (2); hallaremos que, 0 bien x 2

1 +-JS 2'

tg~>--

a,}

2

(3)

o bien,

2

I-J5

x

2

2

-- .

=

cos 3x + 3 cos x , 4

3

sen x

=

3 sen x - sen 3x . 4

(1)

Puesto que, 12 + t + 1 > 0 para todos los val ores reales de t, Ja desigualdad (J) es equivalente a la desigualdad

Valiendonos de estas formulas, escribamos la desigualdad dada en la forma 5 (cos 3x + 3 cos x )cos 3x - (3 sen x - sen 3x )sen 3x > ~ 2

384

PROBLEMAS DE MATEMATICAS ELEMENTALES

o bien

637. Haciendo tg x

sen 2 3x + cos 2 3x + 3 (cos 3x cos x - sen 3x sen x) > ~, 2 o bien

1 cos 4x >-, 2

de donde 1t

1t

3

3

1t

1 2

1t

1 2

- - + -1tn < x < - + -1tn 12

12

cotg~ > 2

2

sen 2

2t tg2x=--2 ' I-t

(n

t2 1 - 3t 2 .

t.

= 0, ± 1, ± 2, ... ).

638. En virtud de que 2 COS 2x cotg x - l = - sen 2 x'

~ + sen